CFAI 3- Economics Flashcards

You may prefer our related Brainscape-certified flashcards:
1
Q

theory of the consumer

A

deals with consumption (the demand for goods and services) by utility-maximizing individuals (i.e., individuals who make decisions that maximize the satisfaction received from present and future consumption).

How well did you know this?
1
Not at all
2
3
4
5
Perfectly
2
Q

theory of the firm

A

deals with the supply of goods and services by profit-maximizing firms. The theory of the consumer and the theory of the firm are important because they help us understand the foundations of demand and supply. Subsequent readings will focus on the theory of the consumer and the theory of the firm.

How well did you know this?
1
Not at all
2
3
4
5
Perfectly
3
Q

Os mercados podem dividir-se em :

A

Factor Markets are markets for the purchase and sale of factors of production. In capitalist private enterprise economies, households own the factors of production (the land, labor, physical capital, and materials used in production)

Good Markets are markets for the output of production. From an economics perspective, firms, which ultimately are owned by individuals either singly or in some corporate form, are organizations that buy the services of those factors. Firms then transform those services into intermediate or final goods and services.

How well did you know this?
1
Not at all
2
3
4
5
Perfectly
4
Q

If, at a given quantity, the highest price that buyers are willing to pay is equal to the lowest price that sellers are willing to accept

A

the quantity is at equilibrium

How well did you know this?
1
Not at all
2
3
4
5
Perfectly
5
Q

when the quantity that buyers are willing and able to purchase at a given price is just equal to the quantity that sellers are willing to offer at that same price, we say the market has discovered ….

A

the equilibrium price

How well did you know this?
1
Not at all
2
3
4
5
Perfectly
6
Q

Qdx=f(Px,I,Py,…)

o que representam as variáveis?

A

where Qdx

represents the quantity demanded of some good X (such as per household demand for gasoline in gallons per week), Px is the price per unit of good X (such as $ per gallon), I is consumers’ income (as in $1,000s per household annually), and Py is the price of another good, Y. (There can be many other goods, not just one, and they can be complements or substitutes.) Equation 1 may be read, “Quantity demanded of good X depends on (is a function of) the price of good X, consumers’ income, the price of good Y, and so on.”

How well did you know this?
1
Not at all
2
3
4
5
Perfectly
7
Q

Qdx=8.4−0.4Px+0.06I−0.01Py

Interpretar a função, sendo que Px é o preço da gasolina, Py é o preço do carro em milhares e I é o income anual em milhares

A

The signs of the coefficients on gasoline price (negative) and consumer’s income (positive) are intuitive, reflecting, respectively, an inverse and a positive relationship between those variables and quantity of gasoline consumed. The negative sign on average automobile price may indicate that if automobiles go up in price, fewer will be purchased and driven; hence less gasoline will be consumed. As will be discussed later, such a relationship would indicate that gasoline and automobiles have a negative cross-price elasticity of demand and are thus complements.

If we pretend to know how does the quantity demanded responds to one variable we have to fix the other variables with a value so we can study the function.

Qdx=8.4−0.4Px+0.06(50)−0.01(20)=11.2−0.4Px

Notice that income and the price of automobiles are not ignored; they are simply held constant, and they are “collected” in the new constant term, 11.2. Notice also that we can rearrange Equation 3, solving for Px in terms of Qx. This operation is called “inverting the demand function,” and gives us Equation 4. (You should be able to perform this algebraic exercise to verify the result.)

Equation (4) 

Px=28−2.5Qx

Equation 4, which gives the per-gallon price of gasoline as a function of gasoline consumed per week, is referred to as the inverse demand function. We need to restrict Qx in Equation 4 to be less than or equal to 11.2 so price is not negative. Henceforward we assume that the reader can work out similar needed qualifications to the valid application of equations. The graph of the inverse demand function is called the demand curve, and is shown in

How well did you know this?
1
Not at all
2
3
4
5
Perfectly
8
Q

Supply Function and the Supply Curve

A

Qsx=f(Px,W,…)

where Qsx

is the quantity supplied of some good X, such as gasoline, Px is the price per unit of good X, and W is the wage rate of labor in, say, dollars per hour. It would be read, “The quantity supplied of good X depends on (is a function of) the price of X (its “own” price), the wage rate paid to labor, etc.”

Equation (8) 

Qsx=−175+250Px−5W

Notice that this supply function says that for every increase in price of $1, this seller would be willing to supply an additional 250 units of the good. Additionally, for every $1 increase in wage rate that it must pay its laborers, this seller would experience an increase in marginal cost and would be willing to supply five fewer units of the good.
We might be interested in the relationship between only two of these variables, price and quantity supplied. Just as we did in the case of the demand function, we use the assumption of ceteris paribus and hold everything except own-price and quantity constant. In our example, we accomplish this by setting W to some value, say, $15. The result is Equation 9:

Equation (9) 

Qsx=−175+250Px−5(15)=−250+250Px

(Institute 10)
Institute, CFA. 2015 CFA Level I Volume 2 Economics. Wiley Global Finance, 2014-07-14. VitalBook file.
A citação disponibilizada é um exemplo. Verifique a exactidão de cada citação antes de utilizar.

How well did you know this?
1
Not at all
2
3
4
5
Perfectly
9
Q

Determine the inverse supply function for an individual seller.

Ou inverse demand function

A

Quando pede a função inversa devemos resolver uma função do tipo
Qsx=−175+250Px−5W
ou
Qdx

em função do tempo com os outros elementos fixados.

How well did you know this?
1
Not at all
2
3
4
5
Perfectly
10
Q

o que nos dá o slope na curva de supply e demmand?

A

a slope de uma função preço/quantidade dá a variação do preço por cada unidade de diferença.

How well did you know this?
1
Not at all
2
3
4
5
Perfectly
11
Q

If the supply curve has negative slope like the demand curve we can say that…

A

Notice that in “Panel A” both demand (D) and supply (S) are negatively sloped, but S is steeper and intersects D from above. In this case, if price is above equilibrium, there will be excess supply and the market mechanism will adjust price downward toward equilibrium. In Panel B, D is steeper, which results in S intersecting D from below. In this case, at a price above equilibrium there will be excess demand, and the market mechanism will dictate that price should rise, thus leading away from equilibrium. This equilibrium would be considered unstable. If price were accidentally displayed above the equilibrium price, the mechanism would not cause price to converge to that equilibrium, but instead to soar above it because there would be excess demand at that price. In contrast, if price were accidentally displayed below equilibrium, the mechanism would force price even further below equilibrium because there would be excess supply.

How well did you know this?
1
Not at all
2
3
4
5
Perfectly
12
Q

Como é que se criam bolhas numa ótica de curva de supply and demand?

A

As a simple approach to understanding bubbles, consider a case in which buyers and sellers base their expectations of future prices on the rate of change of current prices: if price rises, they take that as a sign that price will rise even further. Under these circumstances, if buyers see an increase in price today, they might actually shift the demand curve to the right, desiring to buy more at each price today because they expect to have to pay more in the future. Alternately, if sellers see an increase in today’s price as evidence that price will be even higher in the future, they are reluctant to sell today as they hold out for higher prices tomorrow, and that would shift the supply curve to the left. With a rightward shift in demand and a leftward shift in supply, buyers’ and sellers’ expectations about price are confirmed and the process begins again. This scenario could result in a bubble that would inflate until someone decides that such high prices can no longer be sustained. The bubble bursts and price plunges.

How well did you know this?
1
Not at all
2
3
4
5
Perfectly
13
Q

Diferença entre:

common value auction

private value auction

A

common value auction in which there is some actual common value that will ultimately be revealed after the auction is settled. Prior to the auction’s settlement, however, bidders must estimate that true value. An example of a common value auction would be bidding on a jar containing many coins. Each bidder could estimate the value; but until someone buys the jar and actually counts the coins, no one knows with certainty the true value. In the second case, called a private value auction, each buyer places a subjective value on the item, and in general their values differ. An example might be an auction for a unique piece of art that buyers are hoping to purchase for their own personal enjoyment, not primarily as an investment to be sold later.

Outros tipos de Leilões:

Perhaps the most familiar auction mechanism is the ascending price auction

in which an auctioneer is selling a single item in a face-to-face arena where potential buyers openly reveal their willingness to buy the good at prices that are called out by an auctioneer. The auctioneer begins at a low price and easily elicits nods from buyers. He then raises the price incrementally. In a common value auction, buyers can sometimes learn something about the true value of the item being auctioned from observing other bidders. Ultimately bidders with different maximum amounts they are willing to pay for the item, called reservation prices, begin to drop out of the bidding as price rises above their respective reservation prices.4 Finally, only one bidder is left (who has outbid the bidder with the second highest valuation) and the item is sold to that bidder for his bid price.
Sometimes sellers offer a common value item, such as an oil or timber lease, in a sealed bid auction. In this case, bids are elicited from potential buyers, but there is no ability to observe bids by other buyers until the auction has ended.

In the first price sealed bid auction,

the envelopes containing bids are opened simultaneously and the item is sold to the highest bidder for the actual bid price. Consider an oil lease being auctioned by the government. The highest bidder will pay his bid price but does not know with certainty the profitability of the asset on which he is bidding. The profits that are ultimately realized will be learned only after a successful bidder buys and exploits the asset. Bidders each have some expected value they place on the oil lease, and those values can vary among bidders. Typically, some overly optimistic bidders will value the asset higher than its ultimate realizable value, and they might submit bids above that true value. Because the highest bidder wins the auction and must pay his full bid price, he may find that he has fallen prey to the winner’s curse of having bid more than the ultimate value of the asset. The “winner” in this case will lose money because he has paid more than the value of the asset being auctioned. In recognition of the possibility of being overly optimistic, bidders might bid very conservatively below their expectation of the true value. If all bidders react in this way, the seller might end up with a low sale price.
If the item being auctioned is a private value item, then there is no danger of the winner’s curse (no one would bid more than their own true valuation). But bidders try to guess the reservation prices of other bidders, so the most successful winning bidder would bid a price just above the reservation price of the second-highest bidder. This bid will be below the true reservation price of the highest bidder, resulting in a “bargain” for the highest bidder. To induce each bidder to reveal their true reservation price, sellers can use the second price sealed bid mechanism (also known as a Vickery auction). In this mechanism, the bids are submitted in sealed envelopes and opened simultaneously. The winning buyer is the one who submitted the highest bid, but the price she pays is not equal to her own bid. She pays a price equal to the second-highest bid. The optimal strategy for any bidder in such an auction is to bid her actual reservation price, so the second price sealed bid auction induces buyers to reveal their true valuation of the item. It is also true that if the bidding increments are small, the second price sealed bid auction will yield the same ultimate price as the ascending price auction.
Yet another type of auction is called a descending price auction or Dutch auction in which the auctioneer begins at a very high price—a price so high that no bidder is believed to be willing to pay it.5 The auctioneer then lowers the called price in increments until there is a willing buyer of the item being sold. If there are many bidders, each with a different reservation price and a unit demand, then each has a perfectly vertical demand curve at one unit and a height equal to his reservation price. For example, suppose the highest reservation price is equal to $100. That person would be willing to buy one unit of the good at a price no higher than $100. Suppose each subsequent bidder also has a unit demand and a reservation price that falls, respectively, in increments of $1. The market demand curve would be a negatively sloped step function; that is, it would look like a stair step, with the width of each step being one unit and the height of each step being $1 lower than the preceding step. For example, at a price equal to $90, 11 people would be willing to buy one unit of the good. If the price were to fall to $89, then the quantity demanded would be 12, and so on.

How well did you know this?
1
Not at all
2
3
4
5
Perfectly
14
Q

consumer surplus

A

To get an intuitive feel for this concept, consider the last thing you purchased. Maybe it was a cup of coffee, a new pair of shoes, or a new car. Whatever it was, think of how much you actually paid for it. Now contrast that price with the maximum amount you would have been willing to pay for it instead of going without it altogether. If those two numbers are different, we say you received some consumer surplus from your purchase. You received a “bargain” because you were willing to pay more than you had to pay.

Perceber como identificar o consumer surplus dentro da linha de procura

How well did you know this?
1
Not at all
2
3
4
5
Perfectly
15
Q

Total Surplus, como é que podemos ver pelas curvas de procura e oferta se é o produtor ou o consumidor que mais beneficia do total surplus?

QUando é maximizado?

A

Tudo depende da inclinação da curva da procura e da oferta, quanto mais inclinada for a curva maior é a parcela que a entidade à qual se refere a curva adquire do total surplus.

É maximizado quando estamos no preço de equilibrio.

How well did you know this?
1
Not at all
2
3
4
5
Perfectly
16
Q

No mercado em que o preço está em equilíbrio este pode dizer-se que é

A

o custo marginal da unidade do produto.

How well did you know this?
1
Not at all
2
3
4
5
Perfectly
17
Q

Price Ceiling e Price Floor, como funcionam e quais os problemas?

Como afetam os impostos aplicados ao consumidor e ao produtor?

A

Os Price Ceiling, Price Floor e impostos criam Deadweight Loss é perdido total surplus.

How well did you know this?
1
Not at all
2
3
4
5
Perfectly
18
Q

Quando os impostos incidem sobre o vendedor p.e. como é que calculamos o impacto e quem sai mais afetado?

A

Para calcular o impacto tempos que ver a variação do preço no lado do Vendedor e calcular o novo preço de equilibrio. Com esse preço de equilibrio podemos observar qual a parcela que é imputada ao comprador Peq novo-Peq antigo. Ao vendedor é imputado o remanescente.

A curva com maior inclinação é a que tem o maior surplus, mas também é a que sofre o maior impacto no caso de aumento de impostos. Na teoria é indiferente em quem é imputada a taxa.

No caso de a taxa ser aplicada sobre os vendedores devemos somar a taxa à intercepção na formula visto que os vendedores agora querem mais X para vender.

No caso da taxa ser aplicada aos compradores devemos subtrair a taxa na fórmula porque agora os compradores querem um produto X mais barato visto que vão pagar a taxa.

How well did you know this?
1
Not at all
2
3
4
5
Perfectly
19
Q

O que é o conceito de elasticidade nas curvas de procura e oferta?

Que conclusões podemos tirar de diferntes valores de Elasticidade?

A

Recall that when we introduced the concept of a demand function with Equation 1 earlier, we were simply theorizing that quantity demanded of some good, such as gasoline, is dependent on several other variables, one of which is the price of gasoline itself. We referred to the law of demand that simply states the inverse relationship between the quantity demanded and the price. Although that observation is useful, we might want to dig a little deeper and ask, Just how sensitive is quantity demanded to changes in the price of gasoline? Is it highly sensitive, so that a very small rise in price is associated with an enormous fall in quantity, or is the sensitivity only minimal? It might be helpful if we had a convenient measure of this sensitivity.

Exemplo da elasticidade do preço na curva da procura:
Edpx=%ΔQdx%ΔPx

elasticidade é a variação da quantidade procurada de um ativo devido à variação do seu preço, neste caso. (ambos em percentagem)

1) If Ep > 1, Demand is elastic. The percentage change in price will produce a greater percentage in quantity demanded. If the price goes up, then total revenues will go down. If the price goes down, then total revenues willincrease.
2) If Ep

How well did you know this?
1
Not at all
2
3
4
5
Perfectly
20
Q

Numa reta de procura a procura é elástica…. inelástica… e unitáriamente elástica

Uma recta de procura vertical tem ….

Uma recta de procura horizontal tem …

A

acima do ponto médio, abaixo do ponto médio e no ponto médio.

vertical… tem 0 elasticidade perfectly inelastic

Horizontal tem elasticidade infinita é perfeitamente elástica.

A procura é menos elástica quanto mais dificil for arranjar um substituto para o produto em questão.

For a market, the total expenditure by buyers becomes the total revenue to sellers in that market. It follows, then, that if market demand is elastic, a fall in price will result in an increase in total revenue to sellers as a whole, and if demand is inelastic, a fall in price will result in a decrease in total revenue to sellers. Clearly, if the demand faced by any given seller were inelastic at the current price, that seller could increase revenue by increasing its price. Moreover, because demand is negatively sloped, the increase in price would decrease total units sold, which would almost certainly decrease total cost. So no one-product seller would ever knowingly choose to set price in the inelastic range of its demand.

How well did you know this?
1
Not at all
2
3
4
5
Perfectly
21
Q

income elasticity can be negative, positive, or zero

why? não devia ser sempre positiva?

A

Any good with a positive income of elasticity of demand is said to be a normal good. Luxury goods have high income elasticity (greater than one). The proportionate amount of spending for those goods will go up as incomes increase.

The amount spent on some goods decrease as incomes goes up. Such goods are referred to as inferior goods. Examples of inferior goods include margarine (inferior to butter) and bus travel (inferior to owning a vehicle).

How well did you know this?
1
Not at all
2
3
4
5
Perfectly
22
Q

Elasticity of demand Cross-price

Substitutes and complements

A

Cross elasticity of demand relates the percentage change in quantity demanded of a good to the percentage change in price of a substitute or complementary good. Examples of complementary goods would include peanut butter and jelly, and large SUVs and gasoline. The cross elasticity of demand will be positive for a substitute, and negative for a complement; i.e. demand for a substitute (complement) will go up (down), if the price of the substitute (complement) goes up.

Substitutes are defined empirically. If the cross-price elasticity of two goods is positive, they are substitutes, irrespective of whether someone would consider them “similar.”

For substitute goods, an increase in the price of one good would shift the demand curve for the other good upward and to the right. For complements, however, the impact is in the other direction: When the price of one good rises, the quantity demanded of the other good shifts downward and to the left.

How well did you know this?
1
Not at all
2
3
4
5
Perfectly
23
Q

Consumer Theory

A

Consumer choice theory begins with a fundamental model of how consumer preferences and tastes might be represented. It explores consumers’ willingness to trade off between two goods (or two baskets of goods), both of which the consumer finds beneficial. Consumer choice theory then recognizes that to consume a set of goods and services, consumers must purchase them at given market prices and with a limited income. In effect, consumer choice theory first models what the consumer would like to consume, and then it examines what the consumer can consume with limited income. Finally, by superimposing what the consumer would like to do onto what the consumer can do, we arrive at a model of what the consumer would do under various circumstances. Then by changing prices and income, the model develops consumer demand as a logical extension of consumer choice theory.

How well did you know this?
1
Not at all
2
3
4
5
Perfectly
24
Q

Axioms of the Theory of Consumer Choice

Quais são e o que significam? 3

A

Complete Preferences
Given this understanding of consumption bundles, the first assumption we make about a given consumer’s preferences is simply that she is able to make a comparison between any two possible bundles. That is, given bundles A and B, she must be able to say either that she prefers A to B, or she prefers B to A, or she is indifferent between the two. This is the assumption of complete preferences

Transitive Preferences
Second, we assume that when comparing any three distinct bundles, A, B, and C, if A is preferred to B, and simultaneously B is preferred to C, then it must be true that A is preferred to C. This assumption is referred to as the assumption of transitive preferences

Non Satiation
Finally, we usually assume that in at least one of the goods, the consumer could never have so much that she would refuse any more, even if it were free. This assumption is sometimes referred to as the “more is better” assumption or the assumption of non-satiation

.

How well did you know this?
1
Not at all
2
3
4
5
Perfectly
25
Q

What is the Utility Function?

A

utility function of that particular consumer. The single task of that utility function is to translate each basket of goods and services into a number that rank orders the baskets according to our particular consumer’s preferences. The number itself is referred to as the utility of that basket and is measured in utils, which are just quantities of happiness, or well-being, or whatever comes to mind such that more of it is better than less of it.

How well did you know this?
1
Not at all
2
3
4
5
Perfectly
26
Q

Indifference curves what are they?

A

represent our consumer’s preferences graphically, not just mathematically.

How well did you know this?
1
Not at all
2
3
4
5
Perfectly
27
Q

Marginal rate of substitution MRSxy

A

We capture this willingness to give up one good to obtain a little more of the other
p.e. só aceito trocar uma unidade de vinho por 2 de pão, se continuar a ter cada vez menos vinho e mais pão acaba por só trocar 1 vinho por p.e. 10 pães

is the negative of the slope of the tangent to the indifference curve at any given bundle. If, at some point, the slope of the indifference curve had value –2.5, it means that, starting at that particular bundle, our consumer would be willing to sacrifice wine to obtain bread at the rate of 2.5 ounces of wine per slice of bread

MRSxy =2 - por cada 1 x está disposto a dar 2y.

How well did you know this?
1
Not at all
2
3
4
5
Perfectly
28
Q

why does indifference curves of the same individual cannot cross?

A

Two indifference curves for a given individual cannot cross because the transitivity assumption would be violated.

And although for any given individual two indifference curves cannot cross, there is no reason why two indifference curves for two different consumers cannot intersect.

How well did you know this?
1
Not at all
2
3
4
5
Perfectly
29
Q

We can represent this income constraint (or budget constraint) with the following expression:

Pode ser representada graficamente.

A

The budget constraint shows all the combinations of bread and wine that the consumer could purchase with a fixed amount of income, I, paying prices PB and PW, respectively.

Notice that the slope of the budget constraint is equal to –PB /PW, and it shows the amount of wine that Warren would have to give up if he were to purchase another slice of bread. If the price of bread were to rise, the budget constraint would become steeper, pivoting through the vertical intercept. Alternatively, if the price of wine were to rise, the budget constraint would become less steep, pivoting downward through the horizontal intercept. If income were to rise, the entire budget constraint would shift outward, parallel to the original constraint

PBQB+PWQW≤I

PB preço de B
QB quantidade de B
I = inome do agente economico

How well did you know this?
1
Not at all
2
3
4
5
Perfectly
30
Q

Determining the Consumer’s Equilibrium Bundle of Goods

How do we determine consumer equilibrium?

A

Ao intersectar as indiference curves com a recta de Budget constraint podemos seleccionar o ponto onde obtemos o máximo de satisfação. (quanto mais afastada a curva de indiferença melhor)

How well did you know this?
1
Not at all
2
3
4
5
Perfectly
31
Q

Two-Part Tariff Pricing

o que é?

A

The club could extract all of Johnson’s consumer surplus by charging her a monthly membership fee of €18 plus a per-visit price of €2. This is called a two-part tariff because it assesses one price per unit of the item purchased plus a per-month fee (sometimes called an “entry fee”) equal to the buyer’s consumer surplus evaluated at the per-unit price.

Nestes casos o vendedor sabe qual é o consumer surplus e pode vender o bem ao consumidor pelo marginal cost, acrescentando uma tarifa no valor do consumer surplus. Embora esteja a pagar o marginal cost pelo bem já perdeu a margem toda.

How well did you know this?
1
Not at all
2
3
4
5
Perfectly
32
Q

Income and Substitution Effects for an Inferior Good

and normal good

A

When we apply the income adjustment to isolate substitution effect from income effect, we shift the budget constraint back to constraint 3, reducing income sufficiently to place the consumer back on the original indifference curve. As before, the substitution effect is shown as a movement along the original indifference curve from point a to point b. The income effect is, as before, a movement from one indifference curve to the other, as shown by the movement from point b to point c. In this case, however, the income effect partially offsets the substitution effect, causing demand to be less elastic than if the two effects reinforced each other.
We see that for inferior goods, the income effect and the substitution effect are in opposite directions: The decrease in price causes the consumer to buy more, but the income effect tends to mitigate that effect. It’s still true that a decrease in the price of bread represents an increase in real income. But in the case of an inferior good, the increased income causes the consumer to want to buy less of the good, not more. As long as the income effect has a lower magnitude than the substitution effect, the consumer still ends up buying more at the lower price. However, she buys a little less than she would if the good were normal. It is possible, though highly unlikely, for the income effect to have greater magnitude than the substitution effect.

How well did you know this?
1
Not at all
2
3
4
5
Perfectly
33
Q

What are Giffen Goods?

A

Those inferior goods whose income effect is negative and greater in magnitude than the substitution effect are known as Giffen goods.

How well did you know this?
1
Not at all
2
3
4
5
Perfectly
34
Q

Economic profit vs accounting profit

A

Economic Profit
Economic profit is equal to total revenues less both implicit and explicit costs. For a firm to stay in business, both implicit and explicit costs must be covered. If firms are receiving a negative economic profit in a market, they will leave that market. A normal profitrate exactly covers wage costs and the competitive rate of return on capital.

Accounting Profits
Accounting profits are generally higher than economic profits, as they omit certain costs, such as the value of owner-provided labor and the firm’s equity capital.

When calculating “economic profit”, explicit and opportunity costs are taken into account.

Example:
Suppose someone owns and runs a candy store that grosses $20,000 per month and has operating expenses of $14,000 per month. The store owner particularly enjoys socializing with the customers; this aspect of the business provides a comfort to the owner which is worth $2,000 a month to her. The owner could receive $3,000 a month in interest with the capital that is tied up in the store’s inventory. She could earn $5,000 a month at a different job.

An income statement would show an accounting profit of $6,000 a month:

Explicit Revenues $20,000

Accounting Profit $ 6,000

Answer:
The economic profit, which should determine the economic decision, would be calculated as follows:

Explicit Revenues $20,000

Implicit Revenue
(value of socialization) $ 2,000
- - - - - -
Economic Revenues $22,000
Explicit Costs $14,000
Implicit Costs:
Value of owner's labor $5,000
Required rate of return on
inventory investment $3,000
- - - - - -
Economic Profit ($2,000)

From an economic viewpoint, keeping the candy store open does not make sense. The implicit value of enjoying being with the customers is not of sufficient value in comparison to the fact that the store owner could make more money by working elsewhere and employing the capital elsewhere.

, normal profit is the level of accounting profit needed to just cover the implicit opportunity costs ignored in accounting costs.

How well did you know this?
1
Not at all
2
3
4
5
Perfectly
35
Q

Technological vs. Economic Efficiency

A

Technological efficiency relates quantities of inputs to the quantity of output, while economic efficiency relates the dollar value of inputs to the dollar value of output. A firm would be operating with technological efficiency when it produces a certain level of output with the least amount of input. Economic efficiency would be achieved when a certain level of output is produced with the lowest cost of inputs.

Wall Mart vs Continente
Suppose there are two available methods to produce widgets, one that is highly automated with industrial robots, and a mostly manual one that requires significantly more workers. The automated method costs $50,000 per month to produce 1,000 widgets over a monthly period, using three robots and one worker. The manual method costs $40,000 per month to produce 1,000 widgets over the same time period, with 10 workers that have a minimal amount of tools. We can’t say that either method is technologically inefficient - the automated method requires fewer workers, while the manual method requires less capital for the same quantity of output. However, we can say that the manual method is economically efficient, since it produces 1,000 widgets at the lower cost.

How well did you know this?
1
Not at all
2
3
4
5
Perfectly
36
Q

Major factors promoting cost efficiency and customer service within the corporate world include:

A
  1. The threat of takeover - Inefficient corporate management can attract the interest of outsiders, who will try to take over of the corporation with the intent of running the corporation more efficiently, so as to increase shareholder value. The takeover company most likely would remove the current management. The threat of such a takeover gives management an incentive to serve the interests of corporate shareholders.
  2. Competition for capital and customers - Poor management will tend to drive the price of a company’s stock down, which will tend to make raising more capital difficult. An efficient and/or innovative management will tend to cause the price of a company’s stock to go up, which will make raising additional capital easier. The corporation’s products must be competitive, in terms of both price and quality, in order to attract customers. The production of inferior goods will tend to drive customers away, which will decrease corporate revenues. Therefore, competitive forces tend to limit the ability of management to serve their own needs in lieu of stockholder and customer needs.
  3. Management compensation - Compensation can be set up so that management incentives are in line with those of the corporation. For example, a significant amount of executive compensation can be in the form of stock options, which are of value only when a certain stock price is met.
How well did you know this?
1
Not at all
2
3
4
5
Perfectly
37
Q

What is Concentration within an industry

A

Concentration within an industry refers to the degree to which a small number of firms provide a major portion of the industry’s total production. If concentration is low, then the industry is considered to be competitive. If the concentration is high, then the industry will be viewed as oligopolistic or monopolistic. Government agencies such as the U.S. Department of Justice examine concentration within an industry when deciding to approve potential mergers between industry firms.

The most common measure of concentration is the four-firm concentration ratio, which is defined as the percentage of the industry’s output sold by the four largest firms. An industry with a four-firm concentration ratio of forty percent is generally considered to be competitive.

The Herfindahl-Hirschman Index (HHI) calculates concentration ratios by squaring the market share of the fifty largest firms in an industry. The formula can be expressed as follows:

Formula 3.4

HHI = s12 + s22 + s32 + … + sn2

(where sn is the market share of the ith firm).

A monopoly would have the largest possible value - 1002 = 10000. The HHI for a highly fragmented industry would be close to zero. The Justice Department generally considers an industry with an HHI above 1800 to be highly concentrated.

How well did you know this?
1
Not at all
2
3
4
5
Perfectly
38
Q

Modifying Output ( Short Run vs Long Run)

A

The “Short Run”
The short run is a time period so short that the firm cannot alter some production factors (typically these factors include the size and/or number of plants, the technology used, equipment and the management organization). Those factors are sometimes referred to collectively as the “plant”. The firm usually can increase output in the short run by adding variable inputs. Labor is the most common variable input.

The “Long Run”
In the long run, firms have sufficient time to adjust to any and all production factors. Factories can be expanded, shrunk, demolished or built. The firm can leave or enter an industry.

Suppose a car manufacture decides to build a new plant to build SUVs. This would be an example of a decision made in the long run. If that manufacturer decided to expand output by having employees work overtime, then that would be an example of a short-run decision.

Read more: http://www.investopedia.com/exam-guide/cfa-level-1/microeconomics/modifying-output.asp#ixzz3dGwst6AQ
Follow us: @Investopedia on Twitter

How well did you know this?
1
Not at all
2
3
4
5
Perfectly
39
Q

The economic profit of a firm can be due to:

A
  • competitive advantage;
  • exceptional managerial efficiency or skill;
  • difficult to copy technology or innovation (e.g., patents, trademarks, and copyrights);
  • exclusive access to less-expensive inputs;
  • fixed supply of an output, commodity, or resource;
  • preferential treatment under governmental policy;
  • large increases in demand where supply is unable to respond fully over time;
  • exertion of monopoly power (price control) in the market; and
  • market barriers to entry that limit competition
How well did you know this?
1
Not at all
2
3
4
5
Perfectly
40
Q

Economic Rent

A

When a good as a fixed supply and the market price is above the marginal cost of that good, you have an Economic Rent.

The firm has not done anything internally to merit this special reward: It benefits from an increase in demand in conjunction with a supply curve that does not fully adjust with an increase in quantity when price rises.

It happens with innelastic goods (vertical supply line) mainly commodities or land.

How well did you know this?
1
Not at all
2
3
4
5
Perfectly
41
Q

Accounting Profit= ??

A

normal profit + economic profit

How well did you know this?
1
Not at all
2
3
4
5
Perfectly
42
Q

Perfect competition market vs Imperfect competition

A

In perfect competition market the demand curve is horizontal, since we can offer any quantity and the price taken will be always the same, in the imperfect competition the demand curve is negatively sloped, to a greater quantity the price goes down.

How well did you know this?
1
Not at all
2
3
4
5
Perfectly
43
Q

production function

A

Q = f (K, L)
where Q is the quantity of output, K is capital, and L is labor. The inputs are subject to the constraint that K ≥ 0 and L ≥ 0. A more general production function is stated as:

How well did you know this?
1
Not at all
2
3
4
5
Perfectly
44
Q

Profit maximization occurs when

A
  • the difference between total revenue (TR) and total costs (TC) is the greatest;
  • marginal revenue (MR) equals marginal cost (MC); and
  • the revenue value of the output from the last unit of input employed equals the cost of employing that input unit (as later developed in Equation 12).
How well did you know this?
1
Not at all
2
3
4
5
Perfectly
45
Q

long run/short run concepts and implications on real business

A

The time required for long-run adjustments varies by industry. For example, the long run for a small business using very little in the way of technology and physical capital may be less than a year. On the other hand, for a capital-intensive firm, the long run may be more than a decade. However, given enough time, all production factors are variable, which allows the firm to choose an operating size or plant capacity based on different technologies and physical capital. In this regard, costs and profits will differ between the short run and long run.

How well did you know this?
1
Not at all
2
3
4
5
Perfectly
46
Q

The factors that can lead to diseconomies of scale, inefficiencies, and rising costs when a firm increases in size include:

A
  • So large that it cannot be properly managed.
  • Overlap and duplication of business functions and product lines.
  • Higher resource prices because of supply constraints when buying inputs in large quantities.
How well did you know this?
1
Not at all
2
3
4
5
Perfectly
47
Q

incresing cost industry
decreasing cost industry
constant cost industry

A

The cost of production increases with the quantity produced example :oil

The cost of production decreases with the ammount produced

The cost of production is constant with variable quantitites

How well did you know this?
1
Not at all
2
3
4
5
Perfectly
48
Q

produtivity and bennefits?

A

average output per unit of input

The benefits from increased productivity are as follows:
◾Lower business costs, which translate into increased profitability.
◾An increase in the market value of equity and shareholders’ wealth resulting from an increase in profit.
◾An increase in worker rewards, which motivates further productivity increases from labor.

How well did you know this?
1
Not at all
2
3
4
5
Perfectly
49
Q

Important! Market structure can be broken down into four distinct categories …..????

A

We start with the most competitive environment, perfect competition. Unlike some economic concepts, perfect competition is not merely an ideal based on assumptions. Perfect competition is a reality—for example, in several commodities markets, where sellers and buyers have a strictly homogeneous product and no single producer is large enough to influence market prices. Perfect competition’s characteristics are well recognized and its long-run outcome unavoidable. Profits under the conditions of perfect competition are driven to the required rate of return paid by the entrepreneur to borrow capital from investors (so-called normal profit or rental cost of capital). This does not mean that all perfectly competitive industries are doomed to extinction by a lack of profits. On the contrary, millions of businesses that do very well are living under the pressures of perfect competition.

Monopolistic competition is also highly competitive; however, it is considered a form of imperfect competition. Two economists, Edward H. Chamberlin (US) and Joan Robinson (UK), identified this hybrid market and came up with the term because there are strong elements of competition in this market structure and also some monopoly-like conditions. The competitive characteristic is a notably large number of firms, while the monopoly aspect is the result of product differentiation. That is, if the seller can convince consumers that its product is uniquely different from other, similar products, then the seller can exercise some degree of pricing power over the market. A good example is the brand loyalty associated with soft drinks such as Coca-Cola. Many of Coca-Cola’s customers believe that their beverages are truly different from and better than all other soft drinks. The same is true for fashion creations and cosmetics.

The oligopoly market structure is based on a relatively small number of firms supplying the market. The small number of firms in the market means that each firm must consider what retaliatory strategies the other firms will pursue when prices and production levels change. Consider the pricing behavior of commercial airline companies. Pricing strategies and route scheduling are based on the expected reaction of the other carriers in similar markets. For any given route—say, from Paris, France, to Chennai, India—only a few carriers are in competition. If one of the carriers changes its pricing package, others will likely retaliate. Understanding the market structure of oligopoly markets can help in identifying a logical pattern of strategic price changes for the competing firms.

Finally, the least competitive market structure is monopoly. In pure monopoly markets, there are no other good substitutes for the given product or service. There is a single seller, which, if allowed to operate without constraint, exercises considerable power over pricing and output decisions. In most market-based economies around the globe, pure monopolies are regulated by a governmental authority. The most common example of a regulated monopoly is the local electrical power provider. In most cases, the monopoly power provider is allowed to earn a normal return on its investment and prices are set by the regulatory authority to allow that return.

How well did you know this?
1
Not at all
2
3
4
5
Perfectly
50
Q

elasticidade do preço

A

εP = –(% change in QD) ÷ (% change in P)

εP > 1 Demand is elastic
εP = 1 Demand is unitary elastic
εP

How well did you know this?
1
Not at all
2
3
4
5
Perfectly
51
Q

In a monopoly what is the relation between Marginal Revenue and Elasticity

A

For a monopoly, MR = P[1 – 1/Ep].

How well did you know this?
1
Not at all
2
3
4
5
Perfectly
52
Q

Num mercado competitivo, a longo prazo qual é a relação entre o MC e o AC???

A

São iguais! Se o MC for maior que o AC vão entrar novas empresas a competir até se baixar o MC para valores de AC.

How well did you know this?
1
Not at all
2
3
4
5
Perfectly
53
Q

Se uma empresa tenta tirar market share a uma líder via redução de preço o mais provavel de acontecer é…?

A

Que a market share da empresa dominante continue a aumentar, isto porque é a que à partida terá maior capacidade de aguentar preços baixos, sendo que empresas mais pequenas vão sair do mercado primeiro.

How well did you know this?
1
Not at all
2
3
4
5
Perfectly
54
Q

Oligopoly Nash Model

A

In the Nash model, each company considers the other’s reaction in selecting its strategy. In equilibrium, neither company has an incentive to change its strategy. ThetaTech is better off with open architecture regardless of what SigmaSoft decides. Given this choice, SigmaSoft is better off with a proprietary platform. Neither company will change its decision unilaterally.

How well did you know this?
1
Not at all
2
3
4
5
Perfectly
55
Q

Type of market demand curve on oligopolist market??

A

The oligopolist faces two different demand structures, one for price increases and another for price decreases. Competitors will lower prices to match a price reduction, but will not match a price increase. The result is a kinked demand curve.

How well did you know this?
1
Not at all
2
3
4
5
Perfectly
56
Q

Herfindahl-Hirschmann Index =?

A

Sum ( Parcela de mercado ^2 * nº de empresas)
p.e. o mercado tem 2 empresas com 10% market share e uma com 20% o resto tem menos o HHI do top 3 é
0,1^2*2+0,2^2

Desvantagens deste modelo
The Herfindahl-Hirschmann Index does not reflect low barriers to entry that may restrict the market power of companies currently in the market

How well did you know this?
1
Not at all
2
3
4
5
Perfectly
57
Q

CCI Desvantagens

A

não reflete as mergers corretamente

How well did you know this?
1
Not at all
2
3
4
5
Perfectly
58
Q

GDP Expenditure Approach

A

Consumption (C) - These are personal consumption expenditures. They are typically broken down into the following categories: durable goods, non-durable goods, and services.

Investment (I) - This is gross private investment; it is generally broken down into fixed investment and changes in business inventories.

Government (G) - This category includes government spending on items that are “consumed” in the current period, such as office supplies and gasoline; and also capital goods, such as highways, missiles, and dams. Note that transfer payments are not included in GDP, as they are not part of current production.

Net Exports - This is calculated by subtracting a nations imports (M)from exports (X). Imports are goods and services produced outside the country and consumed within, and exports are goods and services produced domestically and sold to foreigners. Note that this number may be negative, which has occurred in the U.S. for the last several years. Net exports for the U.S. were minus $606 billion during calendar year 2004 (as per Bureau of Economic Analysis, U.S. Department of Commerce June 29, 2005 press release).

Formula 4.1

GDP = C + I + G + (X - M)

How well did you know this?
1
Not at all
2
3
4
5
Perfectly
59
Q

GDP Resource Cost/Income Approach

A

Resource Cost/Income Approach
To calculate Gross Domestic Income (GDI), first consider how revenues received for products and services are used:

  1. Pay for the labor used (wages + income of self-employed proprietors)
  2. Pay for the use of fixed resources, such as land and buildings (rent);
  3. Pay a return to capital employed (interest);
  4. Pay for the replenishment of raw material used.

Remaining revenues go to business owners as a residual cash flow, which is used to replenish capital (depreciation), or it becomes a business profit. So with the resource cost/income approach, GDP (or GDI) is calculated as wages, rent, interest and cash flow paid to business owners or organizers of production.

So GDP by resource cost/income approach = wages + self-employment income + Rent + Interest + profits + indirect business taxes + depreciation + net income of foreigners.

Formula 4.2
GDI = wages + self-employment income + Rent + Interest + profits

+ indirect business taxes + depreciation + net income of foreigners

The above formula is probably hard to memorize, so at least try to remember this relationship - GDI = wages + rent + interest + business cash flow

Total GDP figures should be the same by either method of calculation. But in real life, things don’t always work out this way. Official figures usually have a category called “statistical discrepancy”, which is needed to balance out the two approaches.

How well did you know this?
1
Not at all
2
3
4
5
Perfectly
60
Q

The GDP Deflator

A

The GDP deflator is an economic metric that converts output measured at current prices into constant-dollar GDP. This includes prices for business and government goods and services, as well as those purchased by consumers. This calculation shows how much a change in the base year’s GDP relies upon changes in the price level.

If we wish to analyze the impact of price changes throughout an economy, then the GDP deflator is the preferred price index. This is because it does not focus on a fixed basket of goods and services and automatically reflects changes in consumption patterns and/or the introduction of new goods and services.

Real GDP for a given year, in relation to a “base” year, is computed by multiplying the nominal GDP for a given year by the ratio of the GDP price deflator in the base year to the GDP price deflator for the given year.

Example:
Suppose we wish to calculate the real GDP for the year 2001 in terms of 1996 dollars. The value for (note that these values are for illustration purposes only) 1996 price deflator is 100 and the 2001 price deflator is 115. The 2001 GDP in nominal terms is $10 trillion dollars.

Then: Real GDP year 2001 in 1996 dollars =$10 trillion × (100 / 115) = $8.6 trillion

How well did you know this?
1
Not at all
2
3
4
5
Perfectly
61
Q

Alternative Measures of Domestic Income

A

Other than GDP and GNP, there are alternative measures of domestic income, such as national income, personal income and disposable personal income.

National Income
National income is computed by subtracting indirect business taxes, the net income of foreigners, and depreciation from GDP. It represents the income earned by a country’s citizens. National income can also be computed by summing interest, rents, employee compensation (wages and benefits), proprietors’ income and corporate profits.

·Personal income represents income available for personal use. It is computed by making various adjustments to national income. Social insurance taxes and corporate profits are subtracted from national income, while net interest, corporate dividends and transfer payments are added.

·Disposable personal income (or disposable income) is income available to people after taxes; i.e., it is personal income less individual taxes.

How well did you know this?
1
Not at all
2
3
4
5
Perfectly
62
Q

Key labor market indicators include:

A

The Labor Force Participation Rate - This rate is calculated by dividing the number of people in the civilian labor force by the total civilian population of those 16 years old or older.
The Unemployment Rate - This is computed by dividing the number of unemployed by the number of people in the civilian labor force. That number is multiplied by 100 and expressed as a percentage. Part-time workers are considered to be employed.
The Employment/Population Ratio - This ratio is calculated by dividing the number of job-holding civilians who are at least 16 years old by the total number of people in the civilian population within the same age group. This ratio will tend to go higher during economic booms and lower during recessions.

How well did you know this?
1
Not at all
2
3
4
5
Perfectly
63
Q

Changes in real wage rates can be calculate by

A

dividing nominal wages by the GDP deflator. Data for real wages in the United States is also maintained by the Bureau of Labor Statistics.

How well did you know this?
1
Not at all
2
3
4
5
Perfectly
64
Q

Types of unemployment are often broken down as follows:

A

Structural Unemployment - Changes occur in market economies such that demand increases for some jobs skills while other job skills become outmoded and are no longer in demand. For example, the invention of the automobile increased demand for automobile mechanics and decreased demand for farriers (people who shoe horses).

·Frictional Unemployment - This type of unemployment occurs because of workers who are voluntarily between jobs. Some are looking for better jobs. Due to a lack of perfect information, it takes times to search for the better job. Others may be moving to a different geographical location for personal reasons and time must be spent searching for a new position.

·Cyclical Unemployment - This occurs due to downturns in overall business activity.

As previously noted, full employment does not equate to zero unemployment. Some unemployment is normal in a market economy and is actually expected as part of an efficient labor market. Full employment is defined as the level of employment that occurs when unemployment is normal, taking into account structural and frictional factors.

The natural rate of unemployment is that amount of unemployment that occurs naturally due to imperfect information and job shopping. It is the rate of unemployment that is expected when an economy is operating at full capacity. At this time in the U.S., the natural rate of unemployment is considered to be about 5%.

How well did you know this?
1
Not at all
2
3
4
5
Perfectly
65
Q

CPI

A

Consumer Price Index (CPI)
The CPI represents prices paid by consumers (or households). Prices for a basket of goods are compiled for a certain base period. Price data for the same basket of goods is then collected on a monthly basis. This data is used to compare the prices for a particular month with the prices from a different time period.

Example:
The inflation rate is computed by subtracting the CPI of last year’s prices from the CPI value for this year, dividing that difference by last year’s CPI value and then multiplying by 100.

So if the value of the price index for the current year is equal to 165, and last year’s value was 150, the rate would be calculated as:

Inflation rate = ((165 - 150)/150) X100= 10%

CPI Sources of Bias
The CPI is not a perfect measure of inflation. Sources of bias include:

·Quality adjustments - quality of many goods (e.g., cars, computers, and televisions) goes up every year. Although the Bureau of Labor Statistics is now making adjustments for quality improvements, some price increases may reflect quality adjustments that are still counted entirely as inflation.

·New goods - new goods may be introduced that will be hard to compare to older substitutes.

·Substitution - if the price goes up for one good, consumers may substitute another good that provides similar utility. A common example is beef vs. pork. If the price goes up, and the price of pork stays the same, consumers might easily switch to pork. Although the CPI will go higher due to the price increase in beef, many consumers may not be worse off. Also, when prices go up, consumers may effectively not pay the higher prices by switching to discount stores. The CPI surveys do not check to see if consumers are substituting discount or outlet stores.

How well did you know this?
1
Not at all
2
3
4
5
Perfectly
66
Q

Aggregate Output and Income

Aggregate Expenditure

A

The aggregate output of an economy is the value of all the goods and services produced in a specified period of time. The aggregate income of an economy is the value of all the payments earned by the suppliers of factors used in the production of goods and services. Because the value of the output produced must accrue to the factors of production, aggregate output and aggregate income within an economy must be equal.

Aggregate expenditure, the total amount spent on the goods and services produced in the (domestic) economy during the period, must also be equal to aggregate output and aggregate income. However, some of this expenditure may come from foreigners in the form of net exports.1 Thus, aggregate output, aggregate income, and aggregate expenditure all refer to different ways of decomposing the same quantity.

How well did you know this?
1
Not at all
2
3
4
5
Perfectly
67
Q

real gdp calculus with deflator

and inflation with deflators

A

Real GDP = [Nominal GDP/(GDP deflator/100)]

inflation =( current year deflator/past year deflator)-1

How well did you know this?
1
Not at all
2
3
4
5
Perfectly
68
Q

How to calculate Personal income using the national income?

A

Personal income is a broad measure of household income and measures the ability of consumers to make purchases. As such, it is one of the key determinants of consumption spending. Personal income includes all income received by households, whether earned or unearned. It differs from national income in that some of the income earned by the factors of production (indirect business taxes, corporate income taxes, retained earnings) is not received by households and instead goes to the government or business sectors. Similarly, households receive some income from governments (transfer payments, such as social insurance payments, unemployment compensation, and disability payments) that is not earned. Thus, the following adjustments are made to national income in order to derive personal income:

Personalincome=Nationalincome− Indirectbusinesstaxes− Corporateincometaxes−  Undistributedcorporateprofits+ Transferpayments

.

How well did you know this?
1
Not at all
2
3
4
5
Perfectly
69
Q

Personal disposable income (PDI)

and

household saving calculation

A

It is the personal income less personal taxes

household saving is equal to PDI less three items: consumption expenditures, interest paid by consumers to business, and personal transfer payments to foreigners. The corresponding measure of saving for the business sector equals undistributed corporate profits plus the capital consumption allowance.

How well did you know this?
1
Not at all
2
3
4
5
Perfectly
70
Q

How can we calculete the total tax revenue if we have a deficit?

A

We have to check the Gov Total Spending and the tax revenue will be the total spendig less the deficit.

How well did you know this?
1
Not at all
2
3
4
5
Perfectly
71
Q

borrowing/lending by the business sector in 2009.

A

Para tal devemos somar o capital que pretende gastar + lucros não distribuidos, se a diferença com o CAPEX + variação d Inventórios for positiva o negócio é um lender.

How well did you know this?
1
Not at all
2
3
4
5
Perfectly
72
Q

domestic private saving is used or absorbed in one of three ways:

Private Saving

A

investment spending (I), financing government deficits (G – T), and building up financial claims against overseas economies [positive trade balance, (X – M) > 0]. If there is a trade deficit [(X – M) 0] implies that the private sector must save more than it invests [(S – I) > 0] or the country must run a trade deficit [(X – M)

How well did you know this?
1
Not at all
2
3
4
5
Perfectly
73
Q

The IS Curve

A

The final equation is the IS curve. It summarizes combinations of income and the real interest rate at which income and expenditure are equal. Equivalently, it reflects equilibrium in the goods market.

Y= C+I+G+(X-M)

resolver em ordem a Y income com um dado r interest rate e aí podemos saber qual é o valor para cada componente se tiverem Y.

How well did you know this?
1
Not at all
2
3
4
5
Perfectly
74
Q

The LM Curve

and IS curve intersection

A

The IS curve tells us what level of income is consistent with a given level of the real interest rate but does not address the appropriate level of interest rates, nor does it depend on the price level. In order to determine the interest rate and introduce a connection between output and the price level, we must consider supply and demand in the financial markets. To keep the model as simple as possible, we will deal explicitly with demand and supply for only one financial asset: money. All other assets (e.g., stocks and bonds) are implicitly treated as a composite alternative to holding money. In some of the subsequent discussion, however, we will note differential impacts on equity and fixed-income securities.

The final equation is the IS curve. It summarizes combinations of income and the real interest rate at which income and expenditure are equal. Equivalently, it reflects equilibrium in the goods market.

Para obter a curva LM Real money supply= Real Money Demand

How well did you know this?
1
Not at all
2
3
4
5
Perfectly
75
Q

Aggregate Demand Curve Characteristics

A

AD Curve = IS curve + LM curve = GDP

The AD curve will be flatter if
◾investment expenditure is highly sensitive to the interest rate;
◾saving is insensitive to income;
◾money demand is insensitive to interest rates; and
◾money demand is insensitive to income.
The first two conditions directly imply that income will have to move more to induce a large enough change in saving to match the change in investment spending. All else equal, each of the last two conditions implies that a larger change in the interest rate is required to bring money demand in line with money supply. This, in turn, implies a larger change in investment spending and a correspondingly larger change in saving and income.

How well did you know this?
1
Not at all
2
3
4
5
Perfectly
76
Q

Aggregate Demand Curve

A

The aggregate supply curve (AS curve) represents the level of domestic output that companies will produce at each price level. Unlike the demand side, we must distinguish between the short- and long-run AS curves, which differ with respect to how wages and other input prices respond to changes in final output prices. “Long run” and “short run” are relative terms and are necessarily imprecise with respect to calendar time. The “long run” is long enough that wages, prices, and expectations can adjust but not long enough that physical capital is a variable input. Capital and the available technology to use that capital remain fixed. This condition implies a period of at least a few years and perhaps a decade.

How well did you know this?
1
Not at all
2
3
4
5
Perfectly
77
Q

Impact of Factors Shifting Aggregate Demand

A

Stock pricesRightward: Increase in ADHigher consumption

Housing pricesRightward: Increase in ADHigher consumption

Consumer confidenceRightward: Increase in ADHigher consumption

Business confidenceRightward: Increase in ADHigher investment

Capacity utilizationRightward: Increase in ADHigher investment

Government spendingRightward: Increase in ADGovernment spending a component of AD

TaxesLeftward: Decrease in ADLower consumption and investment

Bank reservesRightward: Increase in ADLower interest rate, higher investment and possibly higher consumption

Exchange rate (foreign currency per unit domestic currency)Leftward: Decrease in ADLower exports and higher imports

Global growthRightward: Increase in ADHigher exports

.

How well did you know this?
1
Not at all
2
3
4
5
Perfectly
78
Q

It is important to understand that short-run macroeconomic equilibrium may occur at a level above or below full employment. We consider four possible types of macroeconomic equilibrium:

A

3.4.1. Long-Run Equilibrium
shows the long-run full employment equilibrium for an economy. In this case, equilibrium occurs where the AD curve intersects the SRAS curve at a point on the LRAS curve. Because equilibrium occurs at a point on the LRAS curve, the economy is at potential real GDP. Both labor and capital are fully employed, and everyone who wants a job has one. In the long run, equilibrium GDP is equal to potential GDP.

3.4.2. Recessionary Gap
Cyclical fluctuations in real GDP and prices are caused by shifts in both the AD and SRAS curves. A decline in AD or a leftward shift in the AD curve results in lower GDP and lower prices. Such declines in AD lead to economic contractions, and if such declines drive demand below the economy’s potential GDP, the economy goes into a recession. In Exhibit 22, when aggregate demand falls, the equilibrium shifts from Point A to Point B. Real GDP contracts from Y1 to Y2, and the aggregate price level falls from P1 to P2. Because of the decline in demand, companies reduce their workforce and the unemployment rate rises. The economy is in recession,15 and the recessionary gap is measured as the difference between Y2 and Y1 or the amount by which equilibrium output is below potential GDP.

If these statistics suggest that a recession is caused by a decline in AD, the following conditions are likely to occur: 
◾Corporate profits will decline.
◾Commodity prices will decline.
◾Interest rates will decline.
◾Demand for credit will decline.

3.4.3. Inflationary Gap

Increases in AD lead to economic expansions as real GDP and employment increase. If the expansion drives the economy beyond its production capacity, however, inflation18 will occur. As summarized in Exhibit 18, higher government spending, lower taxes, a more optimistic outlook among consumers and businesses, a weaker domestic currency, rising equity and housing prices, and an increase in the money supply would each stimulate aggregate demand and shift the AD curve to the right. If aggregate supply does not increase to match the increase in AD, a rise in the overall level of prices will result.
In Exhibit 24, an increase in AD will shift the equilibrium level of GDP from Point A to Point B. Real output increases from Y1 to Y2, and the aggregate price level rises from P1 to P2. As a result of the increase in aggregate demand, companies increase their production and hire more workers. The unemployment rate declines. Once an economy reaches its potential GDP, however, companies must pay higher wages and other input prices to further increase production. The economy now faces an inflationary gap, measured by the difference between Y2 andY1 in Exhibit 24. An inflationary gap occurs when the economy’s short-run level of equilibrium GDP is above potential GDP, resulting in upward pressure on prices.

If economic statistics (consumer sentiment, factory orders for durable and nondurable goods, etc.) suggest that there is an expansion caused by an increase in AD, the following conditions are likely to occur: 
◾Corporate profits will rise.
◾Commodity prices will increase.
◾Interest rates will rise.
◾Inflationary pressures will build.

3.4.4. Stagflation: Both High Inflation and High Unemployment

Structural fluctuations in real GDP are caused by fluctuations in SRAS. Declines in aggregate supply bring about stagflation—high unemployment and increased inflation. Increases in aggregate supply conversely give rise to high economic growth and low inflation.
Exhibit 25 shows the case of a decline in aggregate supply, perhaps caused by an unexpected increase in basic material and oil prices. The equilibrium level of GDP shifts from Point A to B. The economy experiences a recession as GDP falls from Y1 to Y2, but the price level, instead of falling, rises from P1 to P2. Over time, the reduction in output and employment should put downward pressure on wages and input prices and shift the SRAS curve back to the right, re-establishing full employment equilibrium at Point A. However, this mechanism may be painfully slow. Policymakers may use fiscal and monetary policy to shift the AD curve to the right, as previously discussed, but at the cost of a permanently higher price level at Point C.

On the other hand, an increase in AS (rightward shift of the SRAS curve) due to higher productivity growth or lower labor, raw material, and energy costs is favorable for most asset classes other than commodities.

How well did you know this?
1
Not at all
2
3
4
5
Perfectly
79
Q

Grow in potential GDP factors

A

GrowthinpotentialGDP=Growthintechnology+WL(Growthinlabor)+ WC(Growthincapital

where WL and WC are the relative shares of capital and labor in national income. The capital share is the sum of corporate profits, net interest income, net rental income, and depreciation divided by GDP. The labor share is employee compensation divided by GDP. For the United States, WL and WC are roughly 0.7 and 0.3, respectively.
The growth accounting equation highlights a key point: The contribution of labor and capital to long-term growth depends on their respective shares of national income. For the United States, because labor’s share is higher, an increase in the growth rate of labor will have a significantly larger impact (roughly double) on potential GDP growth than will an equivalent increase in the growth rate of capital.

Outra maneira de medir o crescimento potencial do GDP:

The problem with this approach is that there are no observed data on potential GDP or on total factor productivity and both must be estimated. In addition, data on the capital stock and the labor and capital shares of national income are not available for many countries, especially the developing countries.
As an alternative, we can focus on the productivity of the labor force, where we generally have more reliable data. Labor productivity is defined as the quantity of goods and services (real GDP) that a worker can produce in one hour of work. Our standard of living improves if we produce more goods and services for each hour of work. Labor productivity is calculated as real GDP for a given year divided by the total number of hours worked in that year, counting all workers. We use total hours, rather than the number of workers, to adjust for the fact that not everyone works the same number of hours.

Labor productivity = Real GDP/Aggregate hours

Therefore, we need to understand the forces that make labor more productive. Productivity is determined by the factors that we examined in the preceding section: education and skill of workers (human capital), investments in physical capital, and improvements in technology. An increase in any of these factors will increase the productivity of the labor force. The factors determining labor productivity can be derived from the production functions under the assumption of constant returns to scale, where a doubling of inputs causes output to double as well.

Sources of grow
•Labor supply;
•Human capital;
•Physical capital;
•Technology; and
•Natural resources.
How well did you know this?
1
Not at all
2
3
4
5
Perfectly
80
Q

Total Factor Productivity

A

Total factor productivity (TFP) is the component of productivity that proxies technological progress and organizational innovation. TFP is the amount by which output would rise because of improvements in the production process. It is calculated as a residual, the difference between the growth rate of potential output and the weighted average growth rate of capital and labor. Specifically,

TFPgrowth=GrowthinpotentialGDP− [WL(Growthinlabor)+WC(Growthincapital)]

How well did you know this?
1
Not at all
2
3
4
5
Perfectly
81
Q

Rate of sustainable Growth of the Economy

A

Labor productivity data can be used to estimate the rate of sustainable growth of the economy. A useful way to describe potential GDP is as a combination of aggregate hours worked and the productivity of those workers:
Potential GDP = Aggregate hours worked × Labor productivity
Transforming the above equation into growth rates, we get the following:

Potentialgrowthrate=Long-termgrowthrateoflaborforce+ Long-termlaborproductivitygrowthrate

Thus, potential growth is a combination of the long-term growth rate of the labor force and the long-term growth rate of labor productivity. Therefore, if the labor force is growing at 1 percent per year and productivity per worker is rising at 2 percent per year, then potential GDP (adjusted for inflation) is rising at 3 percent per year.

How well did you know this?
1
Not at all
2
3
4
5
Perfectly
82
Q

The sustainable growth rate is best estimated as:

A

growth in the labor force plus growth of labor productivity.

TFP Total Factor Productivity = Potencial GDP -WL -WC
parcela do potencial gdp que depende da tecnologia

How well did you know this?
1
Not at all
2
3
4
5
Perfectly
83
Q

Business Cycle phases

A

A business cycle consists of four phases: trough, expansion, peak, contraction. The period of expansion occurs after the trough (lowest point) of a business cycle and before its peak (highest point), and contraction is the period after the peak and before the trough.1 During the expansion phase, aggregate economic activity is increasing (aggregate is used because some individual economic sectors may not be growing). The contraction—often called a recession, but may be called a depression when exceptionally severe—is a period in which aggregate economic activity is declining (although some individual sectors may be growing). Business cycles are usually viewed as fluctuations around the trend growth of an economy, so such points as peaks and troughs are relative to the individual cycle.

How well did you know this?
1
Not at all
2
3
4
5
Perfectly
84
Q

Capital Spending Phases

A

1- Cortes acima do necessário, começam por equipamentos mais leves, contratações etc os mais pesados vêm no fim porque demoram mais a cancelar.

2-Começam a haver melhorias nas ordens e alguns investimentos que tinham sido cancelados em excesso começam a ser reatados

3-A procura é forte e as fábricas começam a fazer grandes investimentos em novas instalações, etc…

How well did you know this?
1
Not at all
2
3
4
5
Perfectly
85
Q

Though a small part of the overall economy, inventories can reflect growth significantly because they:

A

tend to move forcefully up or down

Quando há uma depressão inicialmente os inventários vão crescer enquanto a empresa faz scale back dos inventórios, posteriormente numa segunda fase em que as vendas são superiores aos inventórios para decrescer os inventórios, quando chegamos ao nível ideal e começam a fazer pick up as vendas a empresa começa a aumentar de novo os inventários.

Os inventários geralmente não refletem bem o estado da economia, as vendas são melhores visto que os inventários são controlados

How well did you know this?
1
Not at all
2
3
4
5
Perfectly
86
Q

Importações e exportações como refletem a economia nacional / global

A

As importações refletem mais a economia nacional, maior GDP mais importações.

As exportações não são tão dependentes da economia nacional e dependem mais da economia global.

How well did you know this?
1
Not at all
2
3
4
5
Perfectly
87
Q

Theories of the Business Cycle

A

Neoclassical analysis relied on the concept of general equilibrium—that is, all markets will reach equilibrium because of the “invisible hand, or free market,” and the price will be found for every good at which supply equals demand. All resources are used efficiently based on the principle of marginal cost equaling marginal revenue, and no involuntary unemployment of labor or capital takes place. In practice, because the neoclassical school provides that the invisible hand will reallocate capital and labor so that they will be used to produce whatever consumers want, it does not allow for “fluctuations found in the aggregate economic activity.” If a shock of any origin shifts either the aggregate demand or aggregate supply curve, the economy will quickly readjust and reach its equilibrium via lower interest rates and lower wages.

But Keynes thought that even if workers agreed to accept lower salaries, this situation might exacerbate the crisis by reducing aggregate demand rather than solving it because lower salary expectations would shift aggregate demand left. For example, if wages fell, workers would need to cut back on their spending. This response would cause a further contraction in the demand for all sorts of goods and services, starting from the more expensive items, such as durable goods, and move in a “domino effect” through the economy (the downward spiral of the AD curve continuously shifting left, as mentioned earlier).
Further, Keynes believed there could be circumstances in which lower interest rates would not reignite growth because business confidence or “animal spirit” was too low. As a consequence, Keynes advocated government intervention in the form of fiscal policy. While he accepted the possibility that markets would reach the equilibrium envisioned by Neoclassical and Austrian economists over the long run, he famously quipped that “in the long run, we are all dead;” that is, the human suffering is excessive while waiting for all shocks to be absorbed and for the economy to return to equilibrium.
When crises occur, the government should intervene to keep capital and labor employed by deliberately running a larger fiscal deficit. This intervention would limit the damages of major recessions. Although this concept continues to be a highly politically charged debate, many economists agree that government expenditure can limit the negative effect of major economic crises in the short term. The practical criticisms that are often expressed about Keynesian fiscal policy are:
1.Fiscal deficits mean higher government debt that needs to be serviced and repaid eventually. There is a danger that government finances could move out of control.
2.Keynesian cyclical policies are focused on the short term. In the long run, the economy may come back and the presence of the expansionary policy may cause it to “overheat”—that is, to have unsustainably fast economic growth, which causes inflation and other problems. This result is because of the typical lags involved in expansionary policy taking effect on the economy.
3.Fiscal policy takes time to implement. Quite often, by the time stimulatory fiscal policy kicks in, the economy is already recovering. (Monetary policy determines the available quantities of money and loans in an economy.)
Keynes’ writings did not advocate a continuous presence of the government in the economy, nor did he suggest using economic policy to “fine tune” the business cycle. He only advocated decisive action in case of a serious economic crisis, such as the Great Depression.

(

How well did you know this?
1
Not at all
2
3
4
5
Perfectly
88
Q

Minsky Moment

A

The term Minsky moment has been coined for a point in business cycle when, after individuals become overextended in borrowing to finance speculative investments, people start realizing that something is likely to go wrong and a panic ensues leading to asset sell-offs. The subprime crisis that affected many industrialized countries starting in 2008 has been represented as a “Minsky moment”4 because it came after years in which risk premiums (e.g., the differentials, or spreads, between very risky bonds and very safe bonds) were at historically low levels. Typically, low risk premiums suggest that no adverse events are expected—in other words, investors believe that because the economy and the markets have been enjoying a protracted expansion, there is no reason to worry about the future. As a consequence, many market observers suggest that business cycles are being tamed. This kind of view of the world leads people to underestimate risk, for example, by not doing the appropriate diligent research before granting a loan or before purchasing a security—in a word, complacency.

Monetarist School

Therefore, Monetarists advocate a focus on maintaining steady growth of the money supply, and otherwise a very limited role for government in the economy. Fiscal and monetary policy should be clear and consistent over time, so all economic agents can forecast government actions. In this way, the uncertainty of economic fluctuations would not be increased by any uncertainty about the timing and magnitude of economic policies and their lagged effects.
According to the Monetarist school, business cycles may occur both because of exogenous shocks and because of government intervention. It is better to let aggregate demand and supply find their own equilibrium than to risk causing further economic fluctuations. However, a key part of monetarist thought is that the money supply needs to continue to grow at a moderate rate. If it falls, as occurred in the 1930s, the economic downturn could be severe, whereas if money grows too fast, inflation will follow.

Real Business Cycle Theory

Because, just like the neoclassical models, the initial New Classical models did not include money, they were called real business cycle models (often abbreviated as RBC). Cycles have real causes, such as the aforementioned changes in technology, whereas monetary variables, such as inflation, are assumed to have no effect on GDP and unemployment.7
RBC models of the business cycle conclude that expansions and contractions represent efficient operation of the economy in response to external real shocks. Because the level of economic activity at any time is consistent with maximizing expected utility, the policy recommendation of RBC theory is for government not to intervene in the economy with discretionary fiscal and monetary policy.
Critics of RBC models often focus on the labor market. Because RBC models rely on efficient markets, it follows that unemployment can only be short term: apart from frictional unemployment,8 if markets are efficient, a person who does not have a job can only be a person who does not want to work. If a person is unemployed, in the context of efficient markets, he just needs to lower his wage rate until he finds an employer who hires him. This assumption is logical because if markets are perfectly flexible, all markets must find equilibrium and full employment.

.

How well did you know this?
1
Not at all
2
3
4
5
Perfectly
89
Q

Simple criteria for the financial analyst wondering if a government’s expenditure is excessive

A
  1. Does the government always have a deficit no matter the cyclical phase, or does it have surpluses during economic booms?
  2. Does the government have a deficit because of a defined series of necessary investments that will improve the productivity of the country, or is it spending most of its money in salaries for patronage employees and on infrastructure of questionable uses?
  3. Is the growth rate of debt (government budget deficit as a percentage of GDP) higher than GDP growth? If so, the debt level will not likely be sustainable.

When government expenditures are excessive, inflation often follows. After that, a recession may occur because the central bank takes necessary measures to slow down an overheated economy. That is, if government purchases increase aggregate demand too much, thus causing inflation (expansionary fiscal policy), the central bank will intervene to stop prices from increasing too quickly (tightening or contractionary monetary policy).

How well did you know this?
1
Not at all
2
3
4
5
Perfectly
90
Q

Unemployment frequently lags the cycle because:

Choose

  • it takes time to compile the employment data.
  • businesses are reluctant to dismiss and hire workers.
  • workers must give notice to employers before quitting jobs.
A

•businesses are reluctant to dismiss and hire workers

How well did you know this?
1
Not at all
2
3
4
5
Perfectly
91
Q

In stagflation what should the central bank do?

A

the economy will typically be left to correct itself because no short-term economic policy is thought to be effective.

How well did you know this?
1
Not at all
2
3
4
5
Perfectly
92
Q

price index

Laspeyres index

Paasche index

FisherIndex

A

Laspeyres index

Nº de bens a comparar são constantes no tempo
.
Preço a que obtenho X bens hoje/ Preço a que obtinha X bens no mes passado (exemplo)

Paasche index

Com a diferença que o nº de bens a comparar é o atual, os preços é que se alteram.
preço a que obtenho os bens hoje / preço que custavam no mês passado

P.e. 112/100 =1,12 =12% inflação

FisherIndex
(LI*PI )^0,5 média geométrica dos 2 índices

How well did you know this?
1
Not at all
2
3
4
5
Perfectly
93
Q

CPI and PPI usos possíveis para um e para outro?

A

The PPI reflects the price changes experienced by domestic producers in a country. Because price increases may eventually pass through to consumers, the PPI can influence the future CPI. The items in the PPI include fuels, farm products (such as grains and meat), machinery and equipment, chemical products (such as drugs and paints), transportation equipment, metals, pulp and paper, and so on. These products are usually further grouped by stage-of-processing categories: crude materials, intermediate materials, and finished goods. Similar to the CPI, scope and weights vary among countries. The differences in the weights can be much more dramatic for the PPI than for the CPI because different countries may specialize in different industries. In some countries, the PPI is called the wholesale price index (WPI).

How well did you know this?
1
Not at all
2
3
4
5
Perfectly
94
Q

Headline and Core Inflation

A

Headline inflation refers to the inflation rate calculated based on the price index that includes all goods and services in an economy. Core inflation usually refers to the inflation rate calculated based on a price index of goods and services except food and energy. Policymakers often choose to focus on the core inflation rate when reading the trend in the economy and making economic policies. The reason is that policymakers are trying to avoid overreaction to short-term fluctuations in food and energy prices that may not have a significant impact on future headline inflation.

How well did you know this?
1
Not at all
2
3
4
5
Perfectly
95
Q

Explaining Inflation

Há duas causas principais para a inflação …

A

Economists describe two types of inflation: cost-push, in which rising costs, usually wages, compel businesses to raise prices generally; and demand-pull, in which increasing demand raise prices generally, which then are reflected in a business’s costs as workers demand wage hikes to catch up with the rising cost of living. Whatever the sequence by which prices and costs rise in an economy, the fundamental cause is the same: excessive demands—either for raw materials, finished goods, or labor—that outstrip the economy’s ability to respond. The initial signs appear in the areas with the greatest constraints: the labor market, the commodity market, or in some area of final output. Even before examining particular cost and price measures, practitioners, when considering inflation, look to indicators that might reveal when the economy faces such constraints.

How well did you know this?
1
Not at all
2
3
4
5
Perfectly
96
Q

Velocity of money

A

Some practitioners view the likelihood of inflationary pressure from the vantage point of the ratio of nominal GDP to money supply, commonly called the “velocity of money.” If this ratio remains stable around a constant or a historical trend, they see reason to look for relative price stability. If velocity falls, it could suggest a surplus of money that might have inflationary potential, but much depends on why it has declined. If velocity has fallen because a cyclical correction has brought down the GDP numerator relative to the money denominator, then practitioners view prospects as more likely to lead to a cyclical upswing to reestablish the former relationship than inflationary pressure. If velocity has fallen, however, because of an increase in the money denominator, then inflationary pressure becomes more likely. If velocity rises, financial analysts might be concerned about a shortage of money in the economy and disinflation or deflation.
The 2008–2009 global recession and financial crisis offers an extreme example of these velocity ambiguities. As the global economy slipped into recession, which held back the GDP numerator in velocity measures, central banks, most notably the Federal Reserve in the United States, tried to help financial institutions cope by injecting huge amounts of money into their respective financial systems, raising the velocity denominator. Velocity measures plummeted accordingly. The expectation is that subsequent GDP growth as economies and financial markets heal will bring velocity back to a more normal level and trend. That said, the fear is that the monetary surge will, over the very long run, lead to inflation. For policy makers, this situation has created a very difficult policy choice. On the one side, they need to sustain the supply of money to help their respective economies cope with the after effects of the financial crisis. On the other side, they need ultimately to withdraw any monetary excess to preclude potential inflationary pressures.

How well did you know this?
1
Not at all
2
3
4
5
Perfectly
97
Q

A diffusion index:

  • measures growth.
  • reflects the consensus change in economic indicators.
  • is roughly analogous to the indices used to measure industrial production
A

•reflects the consensus change in economic indicators.

atribui valor de 1 para os indicadores que sobem 0,5 os que ficam estáveis e 0 para os que descem.
Divide pelo nº de indicadores e multiplica por 100

P.e. Michigan Consumer index

How well did you know this?
1
Not at all
2
3
4
5
Perfectly
98
Q

Based on typical labor utilization patterns across the business cycle, productivity (output per hours worked) is most likely to be highest:

  • at the peak of a boom.
  • into a maturing expansion
  • at the bottom of a recession.
A

•at the bottom of a recession.

How well did you know this?
1
Not at all
2
3
4
5
Perfectly
99
Q

What is the most important effect of labor productivity in a cost-push inflation scenario?

A.Rising productivity indicates a strong economy and a bias towards inflation.
B.The productivity level determines the economy’s status relative to its “natural rate of unemployment.”
C.As productivity growth proportionately exceeds wage increases, product price increases are less likely.

A

B.The productivity level determines the economy’s status relative to its “natural rate of unemployment.”

How well did you know this?
1
Not at all
2
3
4
5
Perfectly
100
Q

Which of the following statements best describes fiscal policy? Fiscal policy:
A.is used by governments to redistribute wealth and incomes.
B.is the attempt by governments to balance their budgets from one year to the next.
C.involves the use of government spending and taxation to influence economy activity.

A

C is correct. Note that governments may wish to use fiscal policy to redistribute incomes and balance their budgets, but the overriding goal of fiscal policy is usually to influence a broader range of economic activity.

How well did you know this?
1
Not at all
2
3
4
5
Perfectly
101
Q

In summary, money fulfills three important functions, it:

A
  • acts as a medium of exchange;
  • provides individuals with a way of storing wealth; and
  • provides society with a convenient measure of value and unit of account.
How well did you know this?
1
Not at all
2
3
4
5
Perfectly
102
Q

Money Creation via Fractional Reserve Banking

aka money multiplier, quanto é neste caso?

Se um cliente tem 100 euros e os deposita num banco e a taxa de reservas minima é 10% quanto dinheiro se pode criar com esses 100 euros em créditos?

A
No primeiro 
90
No segundo
81
No terceiro
81-8,1=72,9
.
.
.
Simplificando = 100/0,1= 1000 EUR

Neste caso o X= Money Multiplier , 100*X=1000, X=10

103
Q

narrow money and/or broad money

A

By narrow money, they generally mean the notes and coins in circulation in an economy, plus other very highly liquid deposits. Broad money encompasses narrow money but also includes the entire range of liquid assets that can be used to make purchases.
Because financial systems, practice, and institutions vary from economy to economy, so do definitions of money; thus, it is difficult to make international comparisons. Still, most central banks produce both a narrow and broad measure of money, plus some intermediate ones too. Exhibit 3 shows the money definitions in four economies.

104
Q

theory of money,

A

Equation (2) 
M × V = P × Y
where M is the quantity of money, V is the velocity of circulation of money (the average number of times in a given period that a unit of currency changes hands), P is the average price level, and Y is real output. The expression is really just an accounting identity. Effectively, it says that over a given period, the amount of money used to purchase all goods and services in an economy, M × V, is equal to monetary value of this output, P × Y.

105
Q

O dinheiro pode ter demand baseado principalmente em 3 factores

A

1- Necessidade para fazer transações, é maior quanto maior for a atividade económica.

2- Liquidez por precaução, é maior se for espectável que irá haver alguma despesa no futuro próximo.

3- Liquidez por especulação, o agente económico vai decidir por não investir no imediato se especular que no futuro próximo irá ter uma melhor oportunidade de investimento.

106
Q

Neutrality in the mone supply

A

an increase in the money supply is thought in the long run simply to lead to an increase in the price level while leaving real variables like output and employment unaffected—is known as money neutrality. To see why in the long run money should have a neutral effect on real things, consider the following simple example.

There are two points worth making with regard to money neutrality. First, although the simple kilogram analogy above does suggest that money should not affect real things in the long run, as the British economist Keynes said: “In the long run we are all dead!” In practice, it is very difficult for economists to be sure that money neutrality holds in the long run. And second, we must assume that monetary authorities do believe that the money supply can affect real things in the short run. If they did not, then there would be almost no point to monetary policy.

107
Q

The Fisher Effect

A

The Fisher effect is directly related to the concept of money neutrality. Named after the economist Irving Fisher, the Fisher effect states that the real rate of interest in an economy is stable over time so that changes in nominal interest rates are the result of changes in expected inflation. Thus, the nominal interest rate (Rnom) in an economy is the sum of the required real rate of interest (Rreal) and the expected rate of inflation (πe) over any given time horizon:

Equation (3) 
Rnom = Rreal + πe

According to money neutrality, over the long term the money supply and/or the growth rate in money should not affect Rreal but will affect inflation and inflation expectations.

108
Q

According to the quantity equation of exchange, an increase in the money supply can lead to an:

A.increase in the aggregate price level, regardless of changes in the velocity of circulation of money.

B.increase in the aggregate price level as long as the velocity of circulation of money rises sufficiently to offset the increase in the money supply.

C.increase in the aggregate price level as long as the velocity of circulation of money does not fall sufficiently to offset the increase in the money supply and real output is unchanged.

A

C.increase in the aggregate price level as long as the velocity of circulation of money does not fall sufficiently to offset the increase in the money supply and real output is unchanged.

109
Q

An expansion in the money supply would most likely:

A.lead to a decline in nominal interest rates.

B.lead to an increase in nominal interest rates.

C.reduce the equilibrium amount of money that economic agents would wish to hold.

A

A.lead to a decline in nominal interest rates.

110
Q

Monetary policy tools

A

Open Market Operations
Open market operations involve the purchase and sale of government bonds from and to commercial banks and/or designated market makers.

The Central Bank’s Policy Rate
Suppose that a central bank announces an increase in its official interest rate. Commercial banks would normally increase their base rates at the same time. A commercial bank’s base rate is the reference rate on which it bases lending rates to all other customers. For example, large corporate clients might pay the base rate plus 1 percent on their borrowing from a bank, while the same bank might lend money to a small corporate client at the base rate plus 3 percent. But why would commercial banks immediately increase their base or reference rates just because the central bank’s refinancing rate had increased?
The answer is that commercial banks would not want to have lent at a rate of interest that would be lower than they might be charged by the central bank. Effectively, the central bank can force commercial banks to borrow from it at this rate because it can conduct open market operations that create a shortage of money, forcing the banks to sell bonds to it with a pre-agreed repurchase price (i.e., do a repurchase agreement). The repo rate would be such that the central bank earned the official refinancing rate on the transactions.

Reserve Requirements
The third primary way in which central banks can limit or increase the supply of money in an economy is via their reserve requirements. We have already seen that the money creation process is more powerful the lower the percentage reserve requirement of banks. So, a central bank could restrict money creation by raising the reserve requirements of banks. However, this policy tool is not used much nowadays in developed economies. Indeed, some central banks, such as the Bank of England, do not even set minimum reserve requirements for the banks under their jurisdiction anymore. Changing reserve requirements frequently is disruptive for banks. For example, if a central bank increased the reserve requirements, a bank that was short on reserves might have to cease its lending activities until it had built up the necessary reserves, because deposits would be unlikely to rise quickly enough for the bank to build its reserves in this way. However, reserve requirements are still actively used in many developing countries to control lending—for example in China and in India—and they remain a potential policy tool for those central banks that do not currently use it.

111
Q

To summarize, the central bank’s policy rate works through the economy via any one, and often all, of the following interconnected channels:

A

◾Short-term interest rates;
◾Changes in the values of key asset prices;
◾The exchange rate; and
◾The expectations of economic agents.

112
Q

When the central bank chooses to target a specific value for its exchange rate:
A.it must also target domestic inflation.
B.it must also set targets for broad money growth.
C.conditions in the domestic economy must adapt to accommodate this target.

A

C is correct. The adoption of an exchange rate target requires that the central bank set interest rates to achieve this target. If the target comes under pressure, domestic interest rates may have to rise, regardless of domestic conditions. It may have a “target” level of inflation in mind as well as “targets” for broad money growth, but as long as it targets the exchange rate, domestic inflation and broad money trends must simply be allowed to evolve.

113
Q

liquidity trap

A

Numa liquidity trap a curva da procura por dinheiro torna-se flat. A injeção de dinheiro não afeta as taxas de juro reais e a politica monetária torna-se ineficiente.

114
Q

Principal limitação da politica monetária

A

The ultimate problem for monetary authorities as they try to manipulate the supply of money in order to influence the real economy is that they cannot control the amount of money that households and corporations put in banks on deposit, nor can they easily control the willingness of banks to create money by expanding credit. Taken together, this also means that they cannot always control the money supply. Therefore, there are definite limits to the power of monetary policy

115
Q

Fiscal Policy

A

involve the use of government spending and changing tax revenue to affect a number of aspects of the economy:

◾Overall level of aggregate demand in an economy and hence the level of economic activity.

◾Distribution of income and wealth among different segments of the population.

◾Allocation of resources between different sectors and economic agents.

116
Q

What are automatic stabilizers?

A

Of course, over the course of a business cycle the budget surplus will vary automatically in a countercyclical way. For example, as an economy slows and unemployment rises, government spending on social insurance and unemployment benefits will also rise and add to aggregate demand. This is known as an automatic stabilizer. Similarly, if boom conditions ensue and employment and incomes are high, then progressive income and profit taxes are rising and also act as automatic stabilizers that would reduce the growing budget surplus. The great advantage of automatic stabilizers is that they are indeed automatic, not requiring the identification of shocks to which policymakers must consider a response.

117
Q

Arguments Agaisnt and Pro being concerned about national debt

A

•The arguments against being concerned about national debt (relative to GDP) are as follows:

◾The scale of the problem may be overstated because the debt is owed internally to fellow citizens. This is certainly the case in Japan, where 93 percent is owned by Japanese residents. In the United States and United Kingdom, the figures are 63 percent and 69 percent, respectively. South Korea and Canada have only 7 percent and 5 percent non-resident ownership of government debt, respectively. But Italy has 49 percent.11

◾A proportion of the money borrowed may have been used for capital investment projects or enhancing human capital (e.g., training, education); these should lead to raised future output and tax revenues.

◾Large fiscal deficits require tax changes which may actually reduce distortions caused by existing tax structures.

◾Deficits may have no net impact because the private sector may act to offset fiscal deficits by increasing saving in anticipation of future increased taxes. This argument is known as “Ricardian equivalence” and is discussed in more detail later.

◾If there is unemployment in an economy, then the debt is not diverting activity away from productive uses (and indeed the debt could be associated with an increase in employment).

•The arguments in favour of being concerned are:

◾High levels of debt to GDP may lead to higher tax rates in the search for higher tax revenues. This may lead to disincentives to economic activity as the higher marginal tax rates reduce labour effort and entrepreneurial activity, leading to lower growth in the long run.

◾If markets lose confidence in a government, then the central bank may have to print money to finance a government deficit. This may lead ultimately to high inflation, as evidenced by the economic history of Germany in the 1920s and more recently in Zimbabwe.

◾Government borrowing may divert private sector investment from taking place (an effect known as crowding out); if there is a limited amount of savings to be spent on investment, then larger government demands will lead to higher interest rates and lower private sector investing.

118
Q

Fiscal multiplier

A

The fiscal multiplier is important in macroeconomics because it tells us how much output changes as exogenous changes occur in government spending or taxation. The recipients of the increase in government spending will typically save a proportion 1 – c of each additional dollar of disposable income, where c is the marginal propensity to consume (MPC) this additional income. Ignoring income taxes, we can see that $c will, in turn, be spent by these recipients on more goods and services. The recipients of this $c will themselves spend a proportion c of this additional income (i.e., $c × c, or c-squared). This process continues with income and spending growing at a constant rate of c as it passes from hand to hand through the economy. This is the familiar geometric progression with constant factor c, where 0 1; this is the multiplier. If c = 0.9 (or individuals spend 90 percent of additions to income), then the multiplier = 1/(1 – 0.9) = 10.

FM = 1/(1-c*(1-t))

t= net taxes
c= marginal propensity to spend
119
Q

In a world where Ricardian equivalence holds, governments would most likely prefer to use monetary rather than fiscal policy because under Ricardian equivalence:

A.real interest rates have a more powerful effect on the real economy.

B.the transmission mechanism of monetary policy is better understood.

C.the future impact of fiscal policy changes are fully discounted by economic agents.

A

C is correct. If Ricardian equivalence holds, then economic agents anticipate that the consequence of any current tax cut will be future tax rises, which leads them to increase their saving in anticipation of this so that the tax cut has little effect on consumption and investment decisions. Governments would be forced to use monetary policy to affect the real economy on the assumption that money neutrality did not hold in the short term.

120
Q

Autarky

A

Autarky é um estado em que um país tem o mercado fechado.
Nestes casos a curva de oferta e procura são feitas apenas com a oferta e procura interna.
Caso se abra o mercado a curva da oferta é deslocada para a direita e encontra-se um novo equilíbrio.

121
Q

absolute advantage
And
comparative advantage

A

Absolute advantage é qual dos dois consegue produzir mais de X com um trabalhador.

Comparative advantage mede o custo de oportunidade relativamente a outro bem de produzir a esse bem, tem uma vantagem comparativa se sacrificando o mesmo nº de Y conseguir produzir mais X do que o outro país.

122
Q

Ricardian and Heckscher–Ohlin Models of Comparative Advantage

A

In the Ricardian model of trade, comparative advantage and the pattern of trade are determined by differences in technology between countries. In the Heckscher–Ohlin model of trade, comparative advantage and the pattern of trade are determined by differences in factor endowments between countries. In reality, technology and factor endowments are complementary, not mutually exclusive, determinants of trade patterns.

123
Q

A voluntary export restraint (VER)

A

is a trade barrier under which the exporting country agrees to limit its exports of the good to its trading partners to a specific number of units. The main difference between an import quota and a VER is that the former is imposed by the importer, whereas the latter is imposed by the exporter. The VER allows the quota rent resulting from the decrease in trade to be captured by the exporter (or exporting country), whereas in the case of an import quota there is ambiguity regarding who captures the quota rents.

124
Q
  • FTA.
  • Economic union.
  • Customs union.
  • Common market.
A

There are many different types of regional trading blocs, depending on the level of integration that takes place. Free trade areas (FTA) are one of the most prevalent forms of regional integration in which all barriers to the flow of goods and services among members have been eliminated. However, each country maintains its own polices against non-members. The North American Free Trade Agreement (NAFTA) among the United States, Canada, and Mexico is an example of a FTA. A customs union extends the FTA by not only allowing free movement of goods and services among members but also creating a common trade policy against non-members. In 1947, Belgium, the Netherlands, and Luxemburg (“Benelux”) formed a customs union that became a part of the European Community in 1958. The common market is the next level of economic integration that incorporates all aspects of the customs union and extends it by allowing free movement of factors of production among members. The Southern Cone Common Market (MERCOSUR) of Argentina, Brazil, Paraguay, and Uruguay is an example of a common market.17 An economic union requires an even greater degree of integration. It incorporates all aspects of a common market and in addition requires common economic institutions and coordination of economic policies among members.

125
Q

Trade creation

Trade diversion

A

Trade creation occurs when regional integration results in the replacement of higher-cost domestic production by lower-cost imports from other members. For example, consider two hypothetical countries, Qualor and Vulcan. Qualor produces 10 million shirts annually and imports 2 million shirts from Vulcan, which has a lower cost of production. Qualor has 10 percent tariffs on imports from Vulcan. Qualor and Vulcan then agree to form a customs union. Qualor reduces its production of shirts to 7 million and now imports 11 million shirts from Vulcan. The decline in Qualor’s domestic production (from 10 million to 7 million shirts) is replaced by importing 3 million additional shirts from the low-cost producer, Vulcan. This scenario represents trade creation. The rest of the additional imports (6 million shirts) represent increased consumption by Qualor’s consumers because the price of shirts declines after formation of the custom union.

Trade diversion occurs when lower-cost imports from nonmember countries are replaced with higher-cost imports from members. In the example in the preceding paragraph, suppose Qualor initially imposes a 10 percent tariff on imports from both Vulcan and Aurelia. Aurelia is the lowest-cost producer of shirts, so Qualor initially imports 2 million shirts from Aurelia instead of from Vulcan. Qualor and Vulcan then form a customs union, which eliminates tariffs on imports from Vulcan but maintains a 10 percent tariff on imports from Aurelia. Now trade diversion could occur if the free trade price on imports from Vulcan is lower than the price on imports from Aurelia. Even though Aurelia is the lowest-cost producer, it may be a higher-priced source of imports because of the tariff. If this is the case, then Qualor will stop importing from Aurelia, a non-member, and divert its imports to Vulcan, a member of the RTA. Both trade creation and trade diversion are possible in an RTA. If trade creation is larger than trade diversion, then the net welfare effect is positive. However, there are concerns that this may not always be the case.

126
Q

Balance of payment Account components

A

The major components of the balance of payments are the

◾current account balance, which largely reflects trade in goods and services.

◾capital account balance, which mainly consists of capital transfers and net sales of non-produced, non-financial assets.

◾financial account, which measures net capital flows based on sales and purchases of domestic and foreign financial assets.

127
Q

Current Account

A

Equation (7) 
CA = Sp – I + (T – G – R)
Because (T – G – R) is taxes minus government spending and transfers, it is the government surplus, or put differently, government savings Sg. Equation 7 can therefore be restated as

Sp= Personal savings
I - investment
T - Taxes 
G- Government Spending
R- Net transfers

CA = Sp + Sg – I
A current account deficit tends to result from low private savings, high private investment, a government deficit (Sg

128
Q

Created after WWII, the International Monetary Fund, the World Bank, and the World Trade Organization are the three major international organizations that provide necessary stability to the international monetary system and facilitate international trade and development.

A
  • The IMF’s mission is to ensure the stability of the international monetary system, the system of exchange rates and international payments that enables countries to buy goods and services from each other. The IMF helps to keep country-specific market risk and global systemic risk under control.
  • The World Bank helps to create the basic economic infrastructure essential for creation and maintenance of domestic financial markets and a well-functioning financial industry in developing countries.
  • The World Trade Organization’s mission is to foster free trade by providing a major institutional and regulatory framework of global trade rules without which today’s global multinational corporations would be hard to conceive.
129
Q

Import Tariff Vs QUota and VER

A

An import tariff and an import quota have the same effect on price, production, and trade. With a quota, however, some or all of the revenue that would be raised by the equivalent tariff is instead captured by foreign producers (or the foreign government) as quota rents. Thus, the welfare loss suffered by the importing country is generally greater with a quota.

A voluntary export restraint (VER) is imposed by the exporting country. It has the same impact on the importing country as an import quota from which foreigners capture all of the quota rents.

130
Q

Nominal Exchange Rates

A

For example, a USD/EUR exchange rate of 1.2875 means that 1 euro will buy 1.2875 US dollars (i.e., 1 euro costs 1.2875 US dollars).1 In this case, the euro is the base currency and the US dollar is the price currency. A decrease in this exchange rate would mean that the euro costs less or that fewer US dollars are needed to buy one euro. In other words, a decline in this exchange rate indicates that the USD is appreciating against the EUR or, equivalently, the EUR is depreciating against the USD.
The exchange rates described above are referred to as nominal exchange rates.

131
Q

Real Exchange Rate

A

An equivalent way of viewing the real exchange rate is that it represents the relative price levels in the domestic and foreign countries. Mathematically, we can represent the foreign price level in terms of the domestic currency as:

Foreign price level in domestic currency = Sd/f × Pf

where Sd/f is the spot exchange rate (quoted in terms of the number of units of domestic currency per one unit of foreign currency) and Pf is foreign price level quoted in terms of the foreign currency. We can define the domestic price level, in terms of the domestic currency, as Pd. Hence, the ratio between the foreign and domestic price levels is:

Real exchange rate(d/f) = (Sd/f × Pf)/Pd = Sd/f × (Pf/Pd)

For example, for a British consumer wanting to buy goods made in the Eurozone, the real exchange rate (defined in GBP/EUR terms; note that the domestic currency for the United Kingdom is the price currency, not the base currency) will be an increasing function of the nominal spot exchange rate (GBP/EUR) and the Eurozone price level and a decreasing function of the UK price level. This is written as:

RealexchangerateGBPEUR=SGBPEUR×(CPIeurCPIUK)

132
Q

Spot Transactions rate used

A

Spot transactions involve the exchange of currencies for immediate delivery. For most currencies, this corresponds to “T + 2” delivery, meaning that the exchange of currencies is settled two business days after the trade is agreed to by the two sides of the deal. (One exception is the Canadian dollar, for which spot settlement against the US dollar is on a T + 1 basis.) The exchange rate used for these spot transactions is referred to as the spot exchange rate, and it is the exchange rate that most people refer to in their daily lives (for example, this is the exchange rate usually quoted by the financial press, on the evening news, and so forth).

133
Q

Outright forward contracts

A

(often referred to simply as forwards) are agreements to deliver foreign exchange at a future date at an exchange rate agreed upon today. For example, suppose that a UK-based company expects to receive a payment of 100 million euros in 85 days. Although it could convert these euros to British pounds with a spot transaction (the spot rate would be the GBP/EUR rate in 83 days, because of T + 2 settlement), this future spot rate is currently unknown and represents a foreign exchange risk to the company. The company can avoid this risk by entering into a transaction with a foreign exchange dealer to sell 100 million euros against the British pound for settlement 85 days from today at a rate—the forward exchange rate—agreed upon today.
As such, forward contracts are any exchange rate transactions that occur with currency settlement longer than the usual T + 2 settlement for spot delivery. Each of these contracts requires two specifications: the date at which the currencies are to be exchanged and the exchange rate to be applied on the settlement date. Accordingly, exchange rates for these transactions are called forward exchange rates to distinguish them from spot rates.

134
Q

FX swap

A

consiste numa transação forward e spot transaction

135
Q

The return (in AUD) on the investment proposed by the investment adviser is closest to:
A.5.00 percent.
B.6.00 percent.
C.7.00 percent.

Spot exchange rate (AUD/HKD) 0.1429
One-year HKD interest rate7.00%
One-year forward exchange rate (AUD/HKD)0.1402

A

A is correct. Converting one AUD to HKD in the spot market gives the client (1/0.1429) = HKD7.00. Investing this for one year leads to 7.00 × (1.07) = HKD7.49. Selling this amount of HKD at the forward rate gives 7.49 × 0.1402 = AUD1.05 (rounding to two decimal places). This implies an AUD-denominated return of 5 percent.

136
Q

A dealer based in New York City provides a spot exchange rate quote of 12.4035 MXN/USD to a client in Mexico City. The inverse of 12.4035 is 0.0806.

1.From the perspective of the Mexican client, the most accurate statement is that the:

A.direct exchange rate quotation is equal to 0.0806.
B.direct exchange rate quotation is equal to 12.4035.
C.indirect exchange rate quotation is equal to 12.4035.

A

B is correct. A direct exchange rate uses the domestic currency as the price currency and the foreign currency as the base currency. For an MXN/USD quote, the MXN is the price currency; therefore, the direct quote for the Mexican client is 12.4035 (it costs 12.4035 pesos to purchase 1 US dollar). Another way of understanding a direct exchange rate quote is that it is the price of one unit of foreign currency in terms of your own currency. This purchase of a unit of foreign currency can be thought of as a purchase much like any other you might make; think of the unit of foreign currency as just another item that you might be purchasing with your domestic currency. For example, for someone based in Canada, a liter of milk currently costs about CAD1.25 and USD1 costs about CAD1.03. This direct currency quote uses the domestic currency (the Canadian dollar, in this case) as the price currency and simply gives the price of a unit of foreign currency that is being purchased.

137
Q

Intuição por detrás do calcula da forward rate

A

Hence, we have two alternative investments—both risk-free because both are invested at risk-free interest rates and because any foreign exchange risk was eliminated (hedged) by using a forward rate. Because these two investments are equal in risk characteristics, they must have the same return. Bearing in mind that the currency quoting convention is the number of foreign currency units per single domestic unit (f/d), this relationship can be stated as:

(1+id)=Sf/d(1+if)(1Ff/d)

id- riskless rate (domestic)
if - riskless rate (foreign)
Sf/d - spot indirecta (moeda base é a doméstica)
Ff/d - forward indirecta

This is an arbitrage relationship because it describes two alternative investments (one on either side of the equal sign) that should have equal returns. If they do not, a riskless arbitrage opportunity exists because an investor can sell short the investment with the lower return and invest the funds in the investment with the higher return; the difference between the two returns is pure profit.7

138
Q

Forward Rates cálculos ver as escolhas múltiplaS?

A

Example 6

Reading 21
(Institute 200)
Institute, CFA. 2015 CFA Level I Volume 2 Economics. Wiley Global Finance, 2014-07-14. VitalBook file.
A citação disponibilizada é um exemplo. Verifique a exactidão de cada citação antes de utilizar.

139
Q

The impact of the exchange rate on trade and capital flows can be analyzed from two perspectives.

A
  • The elasticities approach focuses on the effect of changing the relative price of domestic and foreign goods. This approach highlights changes in the composition of spending. The absorption approach focuses on the impact of exchange rates on aggregate expenditure/saving decisions.
  • The elasticities approach leads to the Marshall–Lerner condition, which describes combinations of export and import demand elasticities such that depreciation (appreciation) of the domestic currency will move the trade balance toward surplus (deficit).
  • The idea underlying the Marshall–Lerner condition is that demand for imports and exports must be sufficiently price-sensitive so that an increase in the relative price of imports increases the difference between export receipts and import expenditures.
  • In order to move the trade balance toward surplus (deficit), a change in the exchange rate must decrease (increase) domestic expenditure (also called absorption) relative to income. Equivalently, it must increase (decrease) domestic saving relative to domestic investment.
  • If there is excess capacity in the economy, then currency depreciation can increase output/income by switching demand toward domestically produced goods and services. Because some of the additional income will be saved, income rises relative to expenditure and the trade balance improves.
  • If the economy is at full employment, then currency depreciation must reduce domestic expenditure in order to improve the trade balance. The main mechanism is a wealth effect: A weaker currency reduces the purchasing power of domestic-currency-denominated assets (including the present value of current and future earned income), and households respond by reducing expenditure and increasing saving.
140
Q

A producer’s supply function is given by the equation

Qss=−55+26Ps+1.3Pa

where Qss

is the quantity of steel supplied by the market, Ps is the per unit price of steel, and Pa is the per unit price of aluminum.

7.If the price of aluminum rises, what happens to the steel producer’s supply curve? The supply curve:
A.shifts to the left.
B.shifts to the right.
C.remains unchanged.

8.If the unit price of aluminum is 10, the slope of the supply curve is closest to:
A.0.04.
B.1.30.
C.26.00.

9.Assume the supply side of the market consists of exactly five identical sellers. If the unit price of aluminum is 20, which equation is closest to the expression for the market inverse supply function?
A.Ps=9.6+0.04Qss
B. Ps=1.1+0.008Qss
C.Qss=−145+130Ps

.

A
  • B is correct. The positive coefficient on the unit price of aluminum implies that aluminum is a substitute for steel. Thus, an increase in the price of aluminum implies that more steel can be sold at given price for steel than before, as steel is substituted for aluminum.
  • A is correct. The slope coefficient of Qss

in the inverse supply function is 0.04.

Startwiththesupplyequation: Qss=−55+26Ps+1.3PaInsertPa=10:   =−55+26Ps+1.3(10)=−42+26PsSolveforPs: Ps=1.6+0.04Qss(theinversesupplyfunction)

•B is correct. Start with the equation Qss=−55+26Ps+1.3Pa

. To aggregate for five suppliers, multiply the individual producer’s supply function by 5:

Qss=5(−55+26Ps+1.3Pa)Qss=−275+130Ps+6.5Pa

.

141
Q

Which of the following statements about market equilibrium is most accurate?
A.The difference between quantity demanded and quantity supplied is zero.
B.The demand curve is negatively sloped and the supply curve is positively sloped.
C.For any given pair of market demand and supply curves, only one equilibrium point can exist

A

A is correct. At market equilibrium the quantity demanded just equals quantity supplied, and thus, the difference between the two is zero.

142
Q

Which statement is most likely to be true in a single-price US Treasury bill auction?
A.Only some non-competitive bids would be filled.
B.Bidders at the highest winning yield may only get a portion of their order filled.
C.All bidders at a yield higher than the winning bid would get their entire order filled.

A

B is correct. Non-competitive bids and bidders at lower yields will get their orders filled first. Securities may then not be available to fill demand entirely at the highest winning yield.

143
Q

A wireless phone manufacturer introduced a next-generation phone that received a high level of positive publicity. Despite running several high-speed production assembly lines, the manufacturer is still falling short in meeting demand for the phone nine months after introduction. Which of the following statements is the most plausible explanation for the demand/supply imbalance?
A.The phone price is low relative to the equilibrium price.
B.Competitors introduced next-generation phones at a similar price.
C.Consumer incomes grew faster than the manufacturer anticipated.

A

A is correct. The situation described is one of excess demand because, in order for markets to clear at the given level of quantity supplied, the company would need to raise prices.

144
Q

If an excise tax is paid by the buyer instead of the seller, which of the following statements is most likely to be true? The price (including tax):
A.paid will be higher than if the seller had paid the tax.
B.received will be lower than if the seller had paid the tax.
C.received will be the same as if the seller had paid the tax.

A

C is correct. The trade price should be the same whether the tax is imposed on the buyer or on the seller.

145
Q

•A quota on an imported good below the market-clearing quantity will most likely lead to which of the following effects?
A.The supply curve shifts upward.
B.The demand curve shifts upward.
C.Some of the buyer’s surplus transfers to the seller.

A

•C is correct. A quota will cause excess demand, raising the price of the good and moving it up and to the left along the demand curve. This should shift some of the buyer’s surplus to the seller.

146
Q

•Assume a market demand function is given by the equation
Qd=50−0.75P
where Qd
is the quantity demanded and P is the price. If P equals 10, the value of the consumer surplus is closest to:
A.67.
B.1,205.
C.1,667.

A

•B is correct. We find consumer surplus as the area of the triangle formed by the y (price) axis, the inverse demand curve, and a line segment from the y axis to the inverse demand function at P = 10.

Putthepriceintothedemandequation: Qd=50−0.75(10)Qd=42.5(thisisthebaseofthetriangle)InvertthedemandfunctionbysolvingforP: −0.75P=Qd−50P=−1.33Qd+66.67

Note the price intercept is 66.67. The height of the triangle is 66.67 – 10 = 56.67. The consumer surplus is the area of the triangle above the price of 10 and below the demand curve, with base equal to the quantity of 42.5: 1/2 Base × Height = (1/2)(42.5)(66.7 – 10) = 1,205.

147
Q

•Which of the following best describes producer surplus?
A.Revenue minus variable costs.
B.Revenue minus variable plus fixed costs.
C.The area above the supply curve and beneath the demand curve and to the left of the equilibrium point.

A

•A is correct. Producer surplus is the difference between the total revenue that sellers receive from selling a given amount of a good and the total variable cost of producing that amount.

148
Q

The market demand function for four-year private universities is given by the equation

Qdpr=84−3.1Ppr+0.8I+0.9Ppu

where Qdpr

is the number of applicants to private universities per year in thousands, Ppr is the average price of private universities (in thousands of USD), I is the household monthly income (in thousands of USD), and Ppu is the average price of public (government-supported) universities (in thousands of USD). Assume that Ppr is equal to 38, I is equal to 100, and Ppu is equal to 18.
23.The price elasticity of demand for private universities is closest to:
A.–3.1.
B.–1.9.
C.0.6.

A

B is correct. From the demand function:

ΔQdpr/ΔPpr=−3.1(thecoeffientinfrontofownprice)SolveforQdpr: Qdpr=84−3.1Ppr+0.8I+0.9Ppu=84−3.1(38)+0.8(100)+0.9(18)=62.4

AtPpr=38,priceelasticityofdemand=(ΔQdpr/ΔPpr)(Ppr/Qdpr)=(−3.1)(38/62.4)=−1.9

149
Q

If the cross-price elasticity between two goods is negative, the two goods are classified as:
A.normal.
B.substitutes.
C.complements.

A

C is correct. With complements, consumption goes up or down together. With a negative cross-price elasticity, as the price of one good goes up, the demand for both falls.

150
Q

•Which of the following ranking systems best describes consumer preferences within a utility function?
A.Util.
B.Ordinal.
C.Cardinal.

A

•B is correct. Utility functions only allow ordinal rankings of consumer preferences.

151
Q

•Which of the following statements best explains why indifference curves are generally convex as viewed from the origin?
A.The assumption of non-satiation results in convex indifference curves.
B.The marginal rate of substitution of one good for another remains constant along an indifference curve.
C.The marginal utility gained from one additional unit of a good versus another diminishes the more one has of the first good.

A

•C is correct. The slope of the indifference curve at any point gives the marginal rate of substitution of one good for another. The curve is convex because the marginal value of one good versus another decreases the more one has of the first good.

152
Q

•If a consumer’s marginal rate of substitution of good X for good Y (MRSXY) is equal to 2, then the:
A.consumer is willing to give up 2 units of X for 1 unit of Y.
B.slope of a line tangent to the indifference curve at that point is 2.
C.slope of a line tangent to the indifference curve at that point is –2.

A

•C is correct. The marginal rate of substitution is equal to the negative of the slope of the tangent to the indifference curve at that point, or –2.

153
Q

•In the case of two goods, x and y, which of the following statements is most likely true? Maximum utility is achieved:
A.along the highest indifference curve below the budget constraint line.
B.at the tangency between the highest attainable indifference curve and the budget constraint line.
C.when the marginal rate of substitution is equal to the ratio of the price of good y to the price of good x.

A

•B is correct. Maximum utility is achieved where the highest attainable indifference curve intersects with just one point (the tangency) on the budget constraint line.

154
Q

•For a Giffen good, the:
A.demand curve is positively sloped.
B.substitution effect overwhelms the income effect.
C.income and substitution effects are in the same direction.

A

•A is correct. The income effect overwhelms the substitution effect such that an increase in the price of the good results in greater demand for the good, resulting in a positively sloped demand curve.

155
Q

•Which of the following statements best illustrates the difference between a Giffen good and a Veblen good?
A.The Giffen good alone is an inferior good.
B.The substitution effect for each is in opposite directions.
C.The Veblen good alone has a positively sloped demand curve.

A

•A is correct. Veblen goods are not inferior goods, whereas Giffen goods are. An increase in income for consumers of a Veblen good leads to an increase in the quantity purchased at each price. The opposite is true for a Giffen good.

156
Q

A firm supplying a commodity product in the marketplace is most likely to receive economic rent if:
A.demand increases for the commodity and supply is elastic.
B.demand increases for the commodity and supply is inelastic.
C.supply increases for the commodity and demand is inelastic.

A

B is correct. Economic rent results when a commodity is fixed in supply (highly inelastic) and the market price is higher than what is required to bring the commodity to market. An increase in demand in this circumstance would result in a rising price and increased potential for economic rent

157
Q

The marketing director for a Swiss specialty equipment manufacturer estimates the firm can sell 200 units and earn total revenue of CHF500,000. However, if 250 units are sold, revenue will total CHF600,000. The marginal revenue per unit associated with marketing 250 units instead of 200 units is closest to:
A.CHF 2,000.
B.CHF 2,400.
C.CHF 2,500.

A

A is correct. Marginal revenue per unit is defined as the change in total revenue divided by the change in quantity sold. MR = ΔTR ÷ ΔQ. In this case, change in total revenue equals CHF100,000, and change in total units sold equals 50. CHF100,000 ÷ 50 = CHF2,000.

158
Q

An operator of a ski resort is considering offering price reductions on weekday ski passes. At the normal price of €50 per day, 300 customers are expected to buy passes each weekday. At a discounted price of €40 per day 450 customers are expected to buy passes each weekday. The marginal revenue per customer earned from offering the discounted price is closest to:
A.€20.
B.€40.
C.€50.

A

A is correct. Marginal revenue per unit is defined as the change in total revenues divided by the change in quantity sold. MR = ΔTR ÷ ΔQ. In this case, change in total revenue per day equals €3,000 [(450 x €40) – (300 x €50)], and change in units sold equals 150 (450 – 300). €3,000 ÷ 150 = €20.

159
Q

The short-term breakeven point of production for a firm operating under perfect competition will most likely occur when:
A.price is equal to average total cost.
B.marginal revenue is equal to marginal cost.
C.marginal revenue is equal to average variable costs.

A

A is correct. Under perfect competition, price equals marginal revenue. A firm breaks even when marginal revenue equals average total cost.

160
Q

A profit maximum is least likely to occur when:
A.average total cost is minimized.
B.marginal revenue equals marginal cost.
C.the difference between total revenue and total cost is maximized.

A

A is correct. The quantity at which average total cost is minimized does not necessarily correspond to a profit maximum.

161
Q

A firm is operating beyond minimum efficient scale in a perfectly competitive industry. To maintain long-term viability the most likely course of action for the firm is to:
A.operate at the current level of production.
B.increase its level of production to gain economies of scale.
C.decrease its level of production to the minimum point on the long-run average total cost curve.

A

C is correct. The firm operating at greater than long-run efficient scale is subject to diseconomies of scale. It should plan to decrease its level of production.

162
Q

A firm experiencing an increase in the marginal product of labor employed would most likely:
A.allow an increased number of workers to specialize and become more adept at their individual functions.
B.find that an increase in workers cannot be efficiently matched by other inputs that are fixed such as property, plant, and equipment.
C.find that the supply of skilled workers is limited, and additional workers lack essential skills and aptitudes possessed by the current workforce.

A

A is correct. Adding new workers in numbers sufficient for them to specialize in their roles and functions should increase marginal product of labor.

163
Q

A firm produces handcrafted wooden chairs, employing both skilled craftsmen and automated equipment in its plant. The selling price of a chair is €100. A craftsman earns €900 per week and can produce ten chairs per week. Automated equipment leased for €800 per week can produce ten chairs per week.
26.The marginal revenue product (per week) of hiring an additional craftsman is closest to:
A.€100.
B.€900.
C.€1,000.

A

C is correct. The marginal revenue product is the marginal product of an additional craftsman (10 chairs) times the price per chair (€100). 10 × €100 = €1,000.

164
Q

Suppose a market has 10 suppliers, each of them with 10 percent of the market. What are the concentration ratio and the HHI of the top four firms?
A.Concentration ratio 4 percent and HHI 40
B.Concentration ratio 40 percent and HHI 0.4
C.Concentration ratio 40 percent and HHI 0.04

A

Solution:

C is correct. The concentration ratio for the top four firms is 10 + 10 + 10 + 10 = 40 percent, and the HHI is 0.102 × 4 = 0.01 × 4 = 0.04.

165
Q

A market structure characterized by many sellers with each having some pricing power and product differentiation is best described as:
A.oligopoly.
B.perfect competition.
C.monopolistic competition.

A

C is correct. Monopolistic competition is characterized by many sellers, differentiated products, and some pricing power.

166
Q

•Upsilon Natural Gas, Inc. is a monopoly enjoying very high barriers to entry. Its marginal cost is $40 and its average cost is $70. A recent market study has determined the price elasticity of demand is 1.5. The company will most likely set its price at:
A.$40.
B.$70.
C.$120.

A

•C is correct. Profits are maximized when MR = MC. For a monopoly, MR = P[1 – 1/Ep]. Setting this equal to MC and solving for P:

$40=P[1−1/1.5]=P*0.333P=$120

167
Q

•The demand schedule in a perfectly competitive market is given by P = 93 – 1.5Q (for Q ≤ 62) and the long-run cost structure of each company is:

Total cost:256 + 2Q + 4Q2Average cost:256/Q + 2 + 4QMarginal cost:2 + 8Q

New companies will enter the market at any price greater than:
A.8.
B.66.
C.81.

A

•B is correct. The long-run competitive equilibrium occurs where MC = AC = P for each company. Equating MC and AC implies 2 + 8Q = 256/Q + 2 + 4Q.
Solving for Q gives Q = 8. Equating MC with price gives P = 2 + 8Q = 66. Any price above 66 yields an economic profit because P = MC > AC, so new companies will enter the market.

168
Q

Over time, the market share of the dominant company in an oligopolistic market will most likely:
A.increase.
B.decrease.
C.remain the same.

A

B is correct. The dominant company’s market share tends to decrease as profits attract entry by other companies.

169
Q

One disadvantage of the Herfindahl-Hirschmann Index is that the index:
A.is difficult to compute.
B.fails to reflect low barriers to entry.
C.fails to reflect the effect of mergers in the industry.

A

B is correct. The Herfindahl-Hirschmann Index does not reflect low barriers to entry that may restrict the market power of companies currently in the market.

170
Q

private saving =…

A

S= Investment +(G-T) +(X-M)

G-T = fiscal deficit

X-M = Trade deficit

171
Q

IS Curve and LM Curve

A
  • The IS curve reflects combinations of GDP and the real interest rate such that aggregate income/output equals planned expenditures. The LM curve reflects combinations of GDP and the interest rate such that demand and supply of real money balances are equal.
  • Combining the IS and LM relationships yields the aggregate demand curve.
172
Q

Short and Long run aggregate supply curve

A
  • The short-run aggregate supply curve is upward sloping because higher prices result in higher profits and induce businesses to produce more and laborers to work more. In the short run, some prices are sticky, implying that some prices do not adjust to changes in demand.
  • In the long run, all prices are assumed to be flexible. The long-run aggregate supply curve is vertical because input costs adjust to changes in output prices, leaving the optimal level of output unchanged. The position of the curve is determined by the economy’s level of potential GDP.
173
Q

stagflation é causada pelo q?

A

Stagflation, a combination of high inflation and weak economic growth, is caused by a decline in short-run aggregate supply.

174
Q

sustainable rate of growth in an economy

potential gdp growth

A

Based on a two-factor production function, Potential GDP growth = Growth in TFP + WL (Growth in labor) + WC (Growth in capital), where WL and WC (= 1 – WL) are the shares of labor and capital in GDP.

The sustainable rate of growth in an economy is determined by the growth rate of the labor supply plus the growth rate of labor productivity.

175
Q

Which of the following statements is the most appropriate description of gross domestic product (GDP)?
A.The total income earned by all households, firms, and the government whose value can be verified.
B.The total amount spent on all final goods and services produced within the economy over a given time period.
C.The total market value of resalable and final goods and services produced within the economy over a given time period.

A

1.B is correct. GDP is the total amount spent on all final goods and services produced within the economy over a specific period of time.

176
Q

Which of the following would be included in Canadian GDP for a given year? The market value of:
A.wine grown in Canada by US citizens.
B.electronics made in Japan and sold in Canada.
C.movies produced outside Canada by Canadian film makers.

A

A is correct. Canadian GDP is the total market value of all final goods and services produced in a given time period within Canada. The wine was produced in Canada and counts towards Canadian GDP.

177
Q

•A GDP deflator less than 1 indicates that an economy has experienced:
A.inflation.
B.deflation.
C.stagflation.

A

•B is correct. The GDP Deflator = Nominal GDP/Real GDP. To get a ratio less than 1, real GDP exceeds nominal GDP, which indicates that prices have decreased and, accordingly, deflation has occurred.

178
Q

•The most accurate description of nominal GDP is:
A.a measure of total expenditures at current prices.
B.the value of goods and services at constant prices.
C.a measure to compare one nation’s economy to another.

A

•A is correct. Nominal GDP is defined as the value of goods and services measured at current prices. Expenditure is used synonymously with the value of goods and services since aggregate expenditures must equal aggregate output of an economy.

179
Q

•From the beginning to the ending years of a decade, the annual value of final goods and services for country X increased from €100 billion to €300 billion. Over that time period, the GDP deflator increased from 111 to 200. Over the decade, real GDP for country X increased by approximately:
A.50%.
B.67%.
C.200%.

A

•B is correct. Real GDP in the first year was €100 billion/1.11 = €90 and in the last year it was €300 billion/2.00 = €150. Thus, (€150 – €90)/€90 = 0.67 or 67%.

180
Q

•If the GDP deflator values for 2008 and 2010 were 190 and 212.8, respectively, which of the following best describes the annual growth rate of the overall price level?
A.5.8%.
B.6%.
C.12%.

A

•A is correct: (212.8/190)1/2 – 1 = 0.0583 or 5.8%.

181
Q

•Consider the following data for 2010 for a hypothetical country:

Account name Amount ($ trillions)
Consumption 15
Capital consumption allowance 1,5
Government spending 3,8
Imports 1,7
Gross private domestic Investment 4,0
Exports 1,5

Based only on the data given, the gross domestic product and national income are respectively closest to:
A.21.1 and 20.6.
B.22.6 and 21.1.
C.22.8 and 20.8.

A

•B is correct. GDP = Consumption + Gross private domestic investment + Government Spending + Exports – Imports = 15 + 4 + 3.8 + 1.5 – 1.7 = 22.6. National income = GDP – CCA = 22.6 –1.5 = 21.1

182
Q

•In calculating personal income for a given year, which of the following would not be subtracted from national income?
A.Indirect business taxes.
B.Undistributed corporate profits.
C.Unincorporated business net income.

A

•C is correct. Unincorporated business net income is also known as proprietor’s income and is included in personal income.

183
Q

•Equality between aggregate expenditure and aggregate output implies that the government’s fiscal deficit must equal:
A.Private saving – Investment – Net exports.
B.Private saving – Investment + Net exports.
C.Investment – Private saving + Net exports.

A

•A is correct. The fundamental relationship among saving, investment, the fiscal balance, and the trade balance is S = I + (G – T) + (X – M). This form of the relationship shows that private saving must fund investment expenditures, the government fiscal balance, and net exports (= net capital outflows). Rearranging gives G – T = (S – I) – (X – M). The government’s fiscal deficit (G – T) must be equal to the private sector’s saving/investment balance (S – I) minus net exports.

184
Q

•Because of a sharp decline in real estate values, the household sector has increased the fraction of disposable income that it saves. If output and investment spending remain unchanged, which of the following is most likely?
A.A decrease in the government deficit.
B.A decrease in net exports and increased capital inflow.
C.An increase in net exports and increased capital outflow.

A

•C is correct. The fundamental relationship among saving, investment, the fiscal balance, and the trade balance is S = I + (G – T) + (X – M). Given the levels of output and investment spending, an increase in saving (reduction in consumption) must be offset by either an increase in the fiscal deficit or an increase in net exports. Increasing the fiscal deficit is not one of the choices, so an increase in net exports and corresponding increase in net capital outflows (increased lending to foreigners and/or increased purchases of assets from foreigners) is the correct response.

185
Q

•Which curve represents combinations of income and the real interest rate at which planned expenditure equals income?
A.The IS curve.
B.The LM curve.
C.The aggregate demand curve.

A

•A is correct. The IS curve represents combinations of income and the real interest rate at which planned expenditure equals income.

186
Q

•An increase in the nominal money supply would shift the:
A.IS curve and the LM curve.
B.IS curve and the aggregate demand curve.
C.LM curve and the aggregate demand curve.

A

•C is correct. The LM curve represents combinations of income and the interest rate at which the demand for real money balances equals the supply. For a given price level, an increase in the nominal money supply is also an increase in the real money supply. To increase the demand for real money balances, either the interest must decline or income must increase. Therefore, at each level of the interest rate, income (= expenditure) must increase—a rightward shift of the LM curve. Since the IS curve is downward sloping (higher income requires a lower interest rate), a rightward shift in the LM curve means that the IS and LM curves will intersect at a higher level of aggregate expenditure/income. This implies a higher level of aggregate expenditure at each price level—a rightward shift of the Aggregate Demand curve.

187
Q

•An increase in the price level would shift the:
A.IS curve.
B.LM curve.
C.aggregate demand curve.

A

•B is correct. The LM curve represents combinations of income and the interest rate at which the demand for real money balances equals the supply. For a given nominal money supply, an increase in the price level implies a decrease in the real money supply. To decrease the demand for real money balances, either the interest must increase or income must decrease. Therefore, at each level of the interest rate, income (= expenditure) must decrease—a leftward shift of the LM curve.

188
Q

•If wages were automatically adjusted for changes in the price level, the short-run aggregate supply curve would most likely be:
A.flatter.
B.steeper.
C.unchanged.

A

•B is correct. The slope of the short-run aggregate supply curve reflects the extent to which wages and other input costs adjust to the overall price level. Automatic adjustment of wages would mitigate the impact of price changes on profitability. Hence, firms would not adjust output as much in response to changing output prices—the SRAS curve would be steeper.

189
Q

•The least likely cause of a decrease in aggregate demand is:
A.higher taxes.
B.a weak domestic currency.
C.a fall in capacity utilization.

A

•B is correct. A weak domestic currency will result in an increase in aggregate demand at each price level—a rightward shift in the AD curve. A weaker currency will cause a country’s exports to be cheaper in global markets. Conversely, imports will be more expensive for domestic buyers. Hence, the net exports component of aggregate demand will increase.

190
Q

•Which of the following is most likely to cause the long-run aggregate supply curve to shift to the left?
A.Higher nominal wages.
B.A decline in productivity.
C.A increase in corporate taxes.

A

•B is correct. Productivity measures the efficiency of labor and is the amount of output produced by workers in a given period of time. A decline in productivity implies decreased efficiency. A decline in productivity increases labor costs, decreases profitability and results in lower output at each output price level—a leftward shift in both the short-run and long-run aggregate supply curves.

191
Q

•The most likely outcome when both aggregate supply and aggregate demand increase is:
A.a rise in inflation.
B.higher employment.
C.an increase in nominal GDP.

A

•B is correct. Higher aggregate demand (AD) and higher aggregate supply (AS) raise real GDP and lower unemployment, meaning employment levels increase.

192
Q

•Which of the following is least likely to be caused by a shift in aggregate demand?
A.Stagflation.
B.A recessionary gap.
C.An inflationary gap.

A

•A is correct. Stagflation occurs when output is declining and prices are rising. This is most likely due to a decline in aggregate supply—a leftward shift of the SRAS curve. Depending on the source of the shift, the LRAS may shift too.

193
Q

•Which of the following can be measured directly?
A.Potential GDP.
B.Labor productivity.
C.Total factor productivity.

A

•B is correct. Labor productivity can be directly measured as output/hour.

194
Q

•The sustainable growth rate is best estimated as:
A.the weighted average of capital and labor growth rates.
B.growth in the labor force plus growth of labor productivity.
C.growth in total factor productivity plus growth in the capital-to-labor ratio.

A

•B is correct. Output growth is equal to the growth rate of the labor force plus the growth rate of labor productivity, i.e. output per worker. Unlike total factor productivity, output per worker is observable, so this is the most practical way to approach estimation of sustainable growth.

195
Q

•In the neoclassical or Solow growth model, an increase in total factor productivity reflects an increase in:
A.returns to scale.
B.output for given inputs.
C.the sustainable growth rate.

A

•B is correct. Total factor productivity (TFP) is a scale factor primarily reflecting technology. An increase in TFP means that output increases for any level of factor inputs.

196
Q

An economic forecasting firm has estimated the following equation from historical data based on the neoclassical growth model:
Potential output growth = 1.5 + 0.72 × Growth of labor + 0.28 × Growth of capital

The coefficient on the growth rate of labor (0.72) in this equation is best interpreted as:
A.the labor force participation rate.
B.the marginal productivity of labor.
C.the share of income earned by labor.

A

C is correct. In the standard Solow growth accounting equation, the coefficient on each factor’s growth rate is its share of income.

197
Q

An economic peak is most closely associated with:
A.accelerating inflation.
B.stable unemployment.
C.declining capital spending.

A

A is correct. Inflation is rising at peaks.

198
Q

The Austrian economic school attributes the primary cause of the business cycle to:
A.misguided government intervention.
B.the creative destruction of technological progress.
C.sticky price and wage expectations that exaggerate trends.

A

•A is correct. Austrian economists see monetary policy mistakes as leading to booms and busts.

199
Q

Monetarists favor a limited role for the government because they argue:
A.government policies operate with a lag.
B.firms take time to adjust to systemic shocks to the economy.
C.resource use is efficient with marginal revenue and cost equal.

A

•A is correct. Monetarists caution policy effects can occur long after the need for which they were implemented is no longer an issue.

200
Q

Deflation is most likely to be associated with:
A.a shortage of government revenue.
B.substantial macroeconomic contraction.
C.explicit monetary policy to combat inflation.

A

B is correct. Deflation is connected to a vicious cycle of reduced spending and higher unemployment.

201
Q

•Assuming the base period for 2010 consumption is November and the initial price index is set at 100, then the inflation rate after calculating the December price index as a Laspeyres index is closest to:
A.19.2%.
B.36.4%.
C.61.6%.

A

•A is correct. The Laspeyres index is calculated with these inputs:
◾November consumption bundle: 70 × 0.9 + 60 × 0.6 = 99
◾December consumption bundle: 70 × 1 + 60 × 0.8 = 118
◾December price index: (118/99) × 100 = 119.19
◾Inflation rate: (119.19/100) – 1 = 0.1919 = 19.19%

202
Q

•For the December consumption basket in Exhibit 1, the value of the Paasche index is closest to:
A.116.
B.148.
C.160.

A

•A is correct. The Paasche index uses the current product mix of consumption combined with the variation of prices. So for December, its value is
(120 × 1 + 50 × 0.8)/(120 × 0.9 + 50 × 0.6) = (160/138) × 100 = 115.9

203
Q

Of the following statements regarding the Producer Price Index (PPI), which is the least likely? The PPI:
A.can influence the future CPI.
B.category weights can vary more widely than analogous CPI terms.
C.is used more frequently than CPI as a benchmark for adjusting labor contract payments.

A

C is correct. The CPI is typically used for this purpose, while the PPI is more closely connected to business contracts.

204
Q

What is the most important effect of labor productivity in a cost-push inflation scenario?
A.Rising productivity indicates a strong economy and a bias towards inflation.
B.The productivity level determines the economy’s status relative to its “natural rate of unemployment.”
C.As productivity growth proportionately exceeds wage increases, product price increases are less likely.

A

C is correct. For productivity, or output per hour, the faster that it can grow, the further that wages can rise without putting pressure on business costs per unit of output.

205
Q

If relative to prior values of their respective indicators, the inventory–sales ratio has risen, unit labor cost is stable, and real personal income has decreased, it is most likely that a peak in the business cycle:
A.has occurred.
B.is just about to occur.
C.will occur sometime into the future.

A

A is correct. Both inventory–sales and unit labor costs are lagging indicators that decline somewhat after a peak. Real personal income is a coincident indicator that by its decline shows a slowdown in business activity.

206
Q

keynisian

monetarism

A

keynisian focus Agregate Demand, government should intervene to pump the economy

Monetarists say that the money supply should be secured and growth at a stable constant pace. Gov. should not intevene on the economy.

207
Q

•As the reserve requirement increases, the money multiplier:
A.increases.
B.decreases.
C.remains the same.

A

•B is correct. There is an inverse relationship between the money multiplier and the reserve requirement. The money multiplier is equal to 1 divided by the reserve requirement.

208
Q

•Which is the most accurate statement regarding the demand for money?
A.Precautionary money demand is directly related to GDP.
B.Transactions money demand is inversely related to returns on bonds.
C.Speculative demand is inversely related to the perceived risk of other assets.

A

•A is correct. Precautionary money demand is directly related to GDP. Precautionary money balances are held to provide a buffer against unforeseen events that might require money. Precautionary balances tend to rise with the volume and value of transactions in the economy, and therefore rise with GDP.

209
Q

According to the theory of money neutrality, money supply growth does not affect variables such as real output and employment in:
A.the long run.
B.the short run.
C.the long and short run.

A

A is correct. According to the theory of money neutrality, an increase in the money supply ultimately leads to an increase in the price level and leaves real variables unaffected in the long run.

210
Q

The proposition that the real interest rate is relatively stable is most closely associated with:
A.the Fisher effect.
B.money neutrality.
C.the quantity theory of money.

A

A is correct. The Fisher effect is based on the idea that the real interest rate is relatively stable. Changes in the nominal interest rate result from changes in expected inflation.

211
Q

Central banks would typically be most concerned with costs of:
A.low levels of inflation that are anticipated.
B.moderate levels of inflation that are anticipated.
C.moderate levels of inflation that are not anticipated.

A

C is correct. Low levels of inflation has higher economic costs than moderate levels, all else equal; unanticipated inflation has greater costs than anticipated inflation.

212
Q

Which action is a central bank least likely to take if it wants to encourage businesses and households to borrow for investment and consumption purposes?
A.Sell long-dated government securities.
B.Purchase long-dated government treasuries.
C.Purchase mortgage bonds or other securities.

A

A is correct. Such action would tend to constrict the money supply and increase interest rates, all else equal.

213
Q

A country that maintains a target exchange rate is most likely to have which outcome when its inflation rate rises above the level of the inflation rate in the target country?
A.An increase in short-term interest rates.
B.An increase in the domestic money supply.
C.An increase in its foreign currency reserves.

A

A is correct. Interest rates are expected to rise to protect the exchange rate target.

214
Q

In theory, setting the policy rate equal to the neutral interest rate should promote:
A.stable inflation.
B.balanced budgets.
C.greater employment.

A

A is correct. The neutral rate of interest is that rate of interest that neither stimulates nor slows down the underlying economy. The neutral rate should be consistent with stable long-run inflation.

215
Q

Which of the following is the most likely example of a tool of fiscal policy?
A.Public financing of a power plant.
B.Regulation of the payment system.
C.Central bank’s purchase of government bonds.

A

A is correct. Public financing of a power plant could be described as a fiscal policy tool to stimulate investment.

216
Q

•Given an independent central bank, monetary policy actions are more likely than fiscal policy actions to be:
A.implementable quickly.
B.effective when a specific group is targeted.
C.effective when combating a deflationary economy.

A

•A is correct. Monetary actions may face fewer delays to taking action than fiscal policy, especially when the central bank is independent.

217
Q

•Which statement regarding fiscal policy is most accurate?
A.To raise business capital spending, personal income taxes should be reduced.
B.Cyclically adjusted budget deficits are appropriate indicators of fiscal policy.
C.An increase in the budget surplus is associated with expansionary fiscal policy.

A

•B is correct. Cyclically adjusted budget deficits are appropriate indicators of fiscal policy. These are defined as the deficit that would exist if the economy was at full employment (or full potential output).

218
Q

•The least likely explanation for why fiscal policy cannot stabilize aggregate demand completely is that:
A.private sector behavior changes over time.
B.policy changes are implemented very quickly.
C.fiscal policy focuses more on inflation than on unemployment.

A

•B is correct. Fiscal policy is subject to recognition, action, and impact lags.

219
Q

•Which statement regarding fiscal deficits is most accurate?
A.Higher government spending may lead to higher interest rates and lower private sector investing.
B.Central bank actions that grow the money supply to address deflationary conditions decrease fiscal deficits.
C.According to the Ricardian equivalence, deficits have a multiplicative effect on consumer spending.

A

•A is correct. Government borrowing may compete with private sector borrowing for investment purposes.

220
Q

•Which policy alternative is most likely to be effective for growing both the public and private sectors?
A.Easy fiscal/easy monetary policy.
B.Easy fiscal/tight monetary policy.
C.Tight fiscal/tight monetary policy.

A

•A is correct. If both fiscal and monetary policies are “easy,” then the joint impact will be highly expansionary, leading to a rise in aggregate demand, low interest rates, and growing private and public sectors.

221
Q

Qual é o principal limite da política monetária?

A

Os Bancos Centrais não conseguem controlar o dinheiro que as familias e empresas querem ou não depositar em última instância só podem alterar as taxas. E os bancos também têm que querer emprestar.

222
Q

Ricardian Equivalence qual é o conceito?

A

An economic theory that suggests that when a government tries to stimulate demand by increasing debt-financed government spending, demand remains unchanged. This is because the public will save its excess money in order to pay for future tax increases that will be initiated to pay off the debt. This theory was developed by David Ricardo in the nineteenth century, but Harvard professor Robert Barro would implement Ricardo’s ideas into more elaborate versions of the same concept.

223
Q

Absolute advantage e comparative advantage

A

A country has an absolute advantage in producing a good (or service) if it is able to produce that good at a lower absolute cost or use fewer resources in its production than its trading partner. A country has a comparative advantage in producing a good if its opportunity cost of producing that good is less than that of its trading partner.

In the Ricardian model of trade, comparative advantage and the pattern of trade are determined by differences in technology between countries. In the Heckscher–Ohlin model of trade, comparative advantage and the pattern of trade are determined by differences in factor endowments between countries. In reality, technology and factor endowments are complementary, not mutually exclusive, determinants of trade patterns.

224
Q

Trade Barriers

import quota
import tariff
VER voluntary export restraint

A
  • In a small country, trade barriers generate a net welfare loss arising from distortion of production and consumption decisions and the associated inefficient allocation of resources.
  • Trade barriers can generate a net welfare gain in a large country if the gain from improving its terms of trade (higher export prices and lower import prices) more than offsets the loss from the distortion of resource allocations. However, the large country can only gain if it imposes an even larger welfare loss on its trading partner(s).
  • An import tariff and an import quota have the same effect on price, production, and trade. With a quota, however, some or all of the revenue that would be raised by the equivalent tariff is instead captured by foreign producers (or the foreign government) as quota rents. Thus, the welfare loss suffered by the importing country is generally greater with a quota.
  • A voluntary export restraint is imposed by the exporting country. It has the same impact on the importing country as an import quota from which foreigners capture all of the quota rents.
225
Q

A regional trading bloc is a group of countries who have signed an agreement to reduce and progressively eliminate barriers to trade and movement of factors of production among the members of the bloc.

5 types of regional trading blocks

A

•They may or may not have common trade barriers against those countries that are not members of the bloc. In a Free Trade Area all barriers to the flow of goods and services among members are eliminated, but each
country maintains its own polices against non-members.

  • A customs union extends the FTA by not only allowing free movement of goods and services among members but also creating a common trade policy against non-members.
  • A common market incorporates all aspects of a customs union and extends it by allowing free movement of factors of production among members.
  • An economic union incorporates all aspects of a common market and requires common economic institutions and coordination of economic policies among members.
  • Members of a monetary union adopt a common currency.
226
Q

Which of the following statements best describes the benefits of international trade?
A.Countries gain from exchange and specialization.
B.Countries receive lower prices for their exports and pay higher prices for imports.
C.Absolute advantage is required for a country to benefit from trade in the long term.

A

A is correct. Countries gain from exchange when trade enables each country to receive a higher price for exported goods and/or pay a lower price for imported goods. This leads to more efficient resource allocation and allows consumption of a larger variety of goods.

227
Q

A country has a comparative advantage in producing a good if:
A.it is able to produce the good at a lower cost than its trading partner.
B.its opportunity cost of producing the good is less than that of its trading partner.
C.its opportunity cost of producing the good is more than that of its trading partner.

A

B is correct. Comparative advantage is present when the opportunity cost of producing a good is less than that of a trading partner.

228
Q

Suppose Mexico exports vegetables to Brazil and imports flashlights used for mining from Brazil. The output per worker per day in each country is as follows:

            Flashlights                                     Vegetables Mexico            20                                                 60 Brazil               40                                                  80

Which country has a comparative advantage in the production of vegetables and what is the most relevant opportunity cost?
A.Brazil: 2 vegetables per flashlight.
B.Mexico: 1.5 vegetables per flashlight.
C.Mexico: 1/3 flashlight per vegetable.

A

C is correct. While Brazil has an absolute advantage in the production of both flashlights and vegetables, Mexico has a comparative advantage in the production of vegetables. The opportunity cost of vegetables in Mexico is ⅓ per flashlight, while the opportunity cost of vegetables in Brazil is ½ per flashlight.

229
Q

Suppose three countries produce rulers and pencils with output per worker per day in each country as follows:

                  Rulers                                            Pencils Mexico            20                                                   40 Brazil                 30                                                 90 China                40                                                 160

Which country has the greatest comparative advantage in the production of rulers?
A.China.
B.Brazil.
C.Mexico.

A

•C is correct. Mexico has the lowest opportunity cost to produce an extra ruler. The opportunity cost is 2 pencils per ruler in Mexico, 3 pencils per ruler in Brazil, and 4 pencils per ruler in China.

230
Q

•In the Ricardian trade model, comparative advantage is determined by:
A.technology.
B.the capital-to-labor ratio.
C.the level of labor productivity.

A

•A is correct. In the Ricardian model, comparative advantage is determined by technology differences between countries. Technology determines output per worker in each industry in each country. Differences in technology between countries cause output per worker in each industry to differ between countries. These ratios determine the pattern of comparative advantage.

231
Q

•In the Ricardian trade model, a country captures more of the gains from trade if:
A.it produces all products while its trade partner specializes in one good.
B.the terms of trade are closer to its autarkic prices than to its partner’s autarkic prices.
C.the terms of trade are closer to its partner’s autarkic prices than to its autarkic prices.

A

•C is correct. A country gains if trade increases the price of its exports relative to its imports as compared to its autarkic prices, i.e. the final terms of trade are more favorable than its autarkic prices. If the relative price of exports and imports remains the same after trade opens, then the country will consume the same basket of goods before and after trade opens, and it gains nothing from the ability to trade. In that case, its trade partner will capture all of the gains. Of course, the opposite is true if the roles are reversed. More generally, a country captures more of the gains from trade the more the final terms of trade differ f

232
Q

According to the Heckscher–Ohlin model, when trade opens:
A.the scarce factor gains relative to the abundant factor in each country.
B.the abundant factor gains relative to the scarce factor in each country.
C.income is redistributed between countries but not within each country.

A

B is correct. As a country opens up to trade, it has a favorable impact on the abundant factor, and a negative impact on the scarce factor. This is because trade causes the output mix to change and therefore changes the relative demand for the factors of production. Increased output of the export product increases demand for the factor that is used intensively in its production, while reduced output of the import product decreases demand for the factor used intensively in its production. Because the export (import) product uses the abundant (scarce) factor intensively, the abundant factor gains relative to the scarce factor in each country.

233
Q

Which of the following trade restrictions is likely to result in the greatest welfare loss for the importing country?
A.A tariff.
B.An import quota.
C.A voluntary export restraint.

A

C is correct. With a voluntary export restraint, the price increase induced by restricting the quantity of imports (= quota rent for equivalent quota = tariff revenue for equivalent tariff) accrues to foreign exporters and/or the foreign government.

234
Q

If Brazil and South Africa have free trade with each other, a common trade policy against all other countries, but no free movement of factors of production between them, then Brazil and South Africa are part of a:
A.customs union.
B.common market.
C.free trade area (FTA).

A

A is correct. A customs union extends a free trade area (FTA) by not only allowing free movement of goods and services among members, but also creating common trade policy against non-members. Unlike a more integrated common market, a customs union does not allow free movement of factors of production among members.

235
Q

•Which of the following factors best explains why regional trading agreements are more popular than larger multilateral trade agreements?
A.Minimal displacement costs.
B.Trade diversions benefit members.
C.Quicker and easier policy coordination.

A

•C is correct. Regional trading agreements are politically less contentious and quicker to establish than multilateral trade negotiations (for example, under the World Trade Organization). Policy coordination and harmonization is easier among a smaller group of countries.

236
Q

•The sale of mineral rights would be captured in which of the following balance of payments components?
A.Capital account.
B.Current account.
C.Financial account.

A

•A is correct. The capital account measures capital transfers and sale and purchase of non-produced, non-financial assets such as mineral rights and intangible assets.

237
Q

•Patent fees and legal services are recorded in which of the following balance of payments components?
A.Capital account.
B.Current account.
C.Financial account.

A

•B is correct. The current account measures the flows of goods and services (including income from foreign investments). Patent fees and legal services are both captured in the services sub-account of the current account.

238
Q

•During the most recent quarter, a steel company in South Korea had the following transactions
◾Bought iron ore from Australia for AUD50 million.
◾Sold finished steel to the United States for USD65 million.
◾Borrowed AUD50 million from a bank in Sydney.
◾Received a USD10 million dividend from US subsidiary.
◾Paid KRW550 million to a Korean shipping company.
Which of the following would be reflected in South Korea’s current account balance for the quarter?
A.The loan.
B.The shipping.
C.The dividend.

A

•C is correct. The current account includes income received on foreign investments. The Korean company effectively “exported” the use of its capital during the quarter to its US subsidiary, and the dividend represents payment for those services.

239
Q

•Which of the following most likely contributes to a current account deficit?
A.High taxes.
B.Low private savings.
C.Low private investment.

A

•B is correct. A current account deficit tends to result from low private saving, high private investment, a government deficit, or a combination of the three. Of the choices, only low private savings contributes toward a current account deficit.

240
Q

A decrease in the real exchange rate (quoted in terms of domestic currency per unit of foreign currency) is most likely to be associated with an increase in which of the following?
A.Foreign price level.
B.Domestic price level.
C.Nominal exchange rate.

A

B is correct. The real exchange rate (quoted in terms of domestic currency per unit of foreign currency) is given by:
Real exchange rate(d/f) = Sd/f × (Pf/Pd)

An increase in the domestic price level (Pd) decreases the real exchange rate because it implies an increase in the relative purchasing power of the domestic currency.

241
Q

•What will be the effect on a direct exchange rate quote if the domestic currency appreciates?
A.Increase
B.Decrease
C.No change

A

•B is correct. In the case of a direct exchange rate, the domestic currency is the price currency (the numerator) and the foreign currency is the base currency (the denominator). If the domestic currency appreciates, then fewer units of the domestic currency are required to buy 1 unit of the foreign currency and the exchange rate (domestic per foreign) declines. For example, if sterling (GBP) appreciates against the euro (EUR), then euro–sterling (GBP/EUR) might decline from 0.8650 to 0.8590.

242
Q

•An executive from Switzerland checked into a hotel room in Spain and was told by the hotel manager that 1 EUR will buy 1.2983 CHF. From the executive’s perspective, an indirect exchange rate quote would be:
A.0.7702 EUR per CHF.
B.0.7702 CHF per EUR.
C.1.2983 EUR per CHF.

A

•A is correct. An indirect quote takes the foreign country as the price currency and the domestic country as the base currency. To get CHF—which is the executive’s domestic currency—as the base currency, the quote must be stated as EUR/CHF. Using the hotel manager’s information, the indirect exchange rate is (1/1.2983) = 0.7702.

243
Q

•Over the past month, the Swiss Franc (CHF) has depreciated 12 percent against pound sterling (GBP). How much has the pound sterling appreciated against the Swiss Franc?
A.12%
B.Less than 12%
C.More than 12%

A

C is correct. The appreciation of sterling against the Swiss franc is simply the inverse of the 12% depreciation of the Swiss franc against Sterling: [1/(1 – 0.12)] – 1 = (1/0.88) – 1 = 0.1364, or 13.64%.

244
Q

Ratio Spot rate
CNY/HKD 0.8422
CNY/ZAR 0.9149
CNY/SEK 1.0218

The spot ZAR/HKD cross-rate is closest to:
A.0.9205.
B.1.0864.
C.1.2978.

Another dealer is quoting the ZAR/SEK cross-rate at 1.1210. The arbitrage profit that can be earned is closest to:
A.ZAR 3671 per million SEK traded.
B.SEK 4200 per million ZAR traded.
C.ZAR 4200 per million SEK traded.

A

•A is correct. To get to the ZAR/HKD cross-rate, it is necessary to take the inverse of the CNY/ZAR spot rate and then multiply by the CNY/HKD exchange rate:

ZAR/HKD=(CNY/ZAR)−1×(CNY/HKD)=(1/0.9149)×0.8422=0.9205

•C is correct. The ZAR/SEK cross-rate from the original dealer is (1.0218/0.9149) = 1.1168, which is lower than the quote from the second dealer. To earn an arbitrage profit, a currency trader would buy SEK (sell ZAR) from the original dealer and sell SEK (buy ZAR) to the second dealer. On 1 million SEK the profit would be
SEK 1,000,000 × (1.1210 – 1.1168) = ZAR 4200 

245
Q

•An exchange rate between two currencies has increased to 1.4500. If the base currency has appreciated by 8% against the price currency, the initial exchange rate between the two currencies was closest to:
A.1.3340.
B.1.3426.
C.1.5660.

A

B is correct. The percentage appreciation of the base currency can be calculated by dividing the appreciated exchange rate by the initial exchange rate. In this case, the unknown is the initial exchange rate. The initial exchange is the value of X that satisfies the formula:
1.4500/X = 1.08

Solving for X leads to 1.45/1.08 = 1.3426.

246
Q

•A BRL/MXN spot rate is listed by a dealer at 0.1378. The 6-month forward rate is 0.14193. The 6-month forward points are closest to:
A.–41.3.
B.+41.3.
C.+299.7.

A

•B is correct. The number of forward points equals the forward rate minus the spot rate, or 0.14193 – 0.1378 = 0.00413, multiplied by 10,000: 10,000 × 0.00413= 41.3 points. By convention, forward points are scaled so that ±1 forward point corresponds to a change of ±1 in the last decimal place of the spot exchange rate.

247
Q

•A three-month forward exchange rate in CAD/USD is listed by a dealer at 1.0123. The dealer also quotes 3-month forward points as a percentage at 6.8%. The CAD/USD spot rate is closest to:
A.0.9478.
B.1.0550.
C.1.0862.

A

•A is correct. Given the forward rate and forward points as a percentage, the unknown in the calculation is the spot rate. The calculation is as follows:
Spot rate × (1 + Forward points as a percentage) = Forward rate

Spot rate × (1 + 0.068) = 1.0123

Spot = 1.0123/1.068 =0.9478

248
Q

•If the base currency in a forward exchange rate quote is trading at a forward discount, which of the following statements is most accurate?
A.The forward points will be positive.
B.The forward percentage will be negative.
C.The base currency is expected to appreciate versus the price currency.

A

•B is correct. The base currency trading at a forward discount means that 1 unit of the base currency costs less for forward delivery than for spot delivery; i.e., the forward exchange rate is less than the spot exchange rate. The forward points, expressed either as an absolute number of points or as a percentage, are negative.

249
Q

•A forward premium indicates:
A.an expected increase in demand for the base currency.
B.the interest rate is higher in the base currency than in the price currency.
C.the interest rate is higher in the price currency than in the base currency.

A

•C is correct. To eliminate arbitrage opportunities, the spot exchange rate (S), the forward exchange rate (F), the interest rate in the base currency (ib), and the interest rate in the price currency (ip) must satisfy:

FS=(1+ip1+ib)

According to this formula, the base currency will trade at forward premium (F > S) if, and only if, the interest rate in the price currency is higher than the interest rate in the base currency (ip > ib).

250
Q

•The JPY/AUD spot exchange rate is 82.42, the JPY interest rate is 0.15%, and the AUD interest rate is 4.95%. If the interest rates are quoted on the basis of a 360-day year, the 90-day forward points in JPY/AUD would be closest to:
A.–377.0.
B.–97.7.
C.98.9.

A

•B is correct.

Mucho IMPORTANTÈEEEEEE!!!

Forward/Spot= (1+ip)/(1+ib)

ip = interest da price currency
ib= interest da base currency

The forward points are 100 × (F – S) = 100 × (81.443 – 82.42) = 100 × (–0.977) = –97.7. Note that because the spot exchange rate is quoted with two decimal places, the forward points are scaled by 100.

251
Q

•Which of the following is not a condition of an ideal currency regime?
A.Fully convertible currencies.
B.Fully independent monetary policy.
C.Independently floating exchange rates.

A

•C is correct. An ideal currency regime would have credibly fixed exchange rates among all currencies. This would eliminate currency-related uncertainty with respect to the prices of goods and services as well as real and financial assets.

252
Q

A fixed exchange rate regime in which the monetary authority is legally required to hold foreign exchange reserves backing 100% of its domestic currency issuance is best described as:
A.dollarization.
B.a currency board.
C.a monetary union.

A

•B is correct. With a currency board, the monetary authority is legally required to exchange domestic currency for a specified foreign currency at a fixed exchange rate. It cannot issue domestic currency without receiving foreign currency in exchange, and it must hold that foreign currency as a 100% reserve against the domestic currency issued. Thus, the country’s monetary base (bank reserves plus notes and coins in circulation) is fully backed by foreign exchange reserves.

253
Q

A country with a trade deficit will most likely:
A.have an offsetting capital account surplus.
B.save enough to fund its investment spending.
C.buy assets from foreigners to fund the imbalance.

A

•A is correct. A trade deficit must be exactly matched by an offsetting capital account surplus to fund the deficit. A capital account surplus reflects borrowing from foreigners (an increase in domestic liabilities) and/or selling assets to foreigners (a decrease in domestic assets). A capital account surplus is often referred to as a “capital inflow” because the net effect is foreign investment in the domestic economy.

254
Q

Suppose an investment analyst takes a random sample of US equity mutual funds and calculates the average Sharpe ratio. The sample size is 100, and the average Sharpe ratio is 0.45. The sample has a standard deviation of 0.30. Calculate and interpret the 90 percent confidence interval for the population mean of all US equity mutual funds by using a reliability factor based on the standard normal distribution.
The reliability factor for a 90 percent confidence interval, as given earlier, is z0.05 = 1.65. The confidence interval will be

A

The confidence interval spans 0.4005 to 0.4995, or 0.40 to 0.50, carrying two decimal places. The analyst can say with 90 percent confidence that the interval includes the population mean.
In this example, the analyst makes no specific assumption about the probability distribution describing the population. Rather, the analyst relies on the central limit theorem to produce an approximate normal distribution for the sample mean.